Internal Medicine With Answers

April 27, 2017 | Author: clinicalskills | Category: N/A
Share Embed Donate


Short Description

Download Internal Medicine With Answers...

Description

Assessment Exam for USMLE Step 2: Internal Medicine *

Answers and Explanations

*USMLE is a joint program of the Federation of State Medical Boards of the United States, Inc. and the National Board of Medical Examiners.

©2003 Kaplan, Inc. All rights reserved. No part of this book may be reproduced in any form, by photostat, microfilm, xerography or any other means, or incorporated into any information retrieval system, electronic or mechanical, without the written permission of Kaplan, Inc. Not for resale.

Internal Medicine Assessment Exam 1.

A 47-year-old man returns to the physician for follow-up of high blood pressure. On the last visit, the diagnosis of hypertension was confirmed, and he was started on a low dose of felodipine. Today he is without complaint and denies any abnormal weight loss or gain, muscle weakness, or skin changes. A full review of systems is unrevealing. His past medical history is remarkable for depression with sleep disturbance, for which he takes fluoxetine. His blood pressure is 165/79 mm Hg, and pulse is 78/min. Physical examination is otherwise unremarkable. Laboratory studies today show: Serum Sodium

144 mEq/L

Potassium

2.9 mEq/L

Chloride

105 mEq/L

Bicarbonate

27 mEq/L

Blood urea nitrogen

18 mg/dL

Creatinine

0.9 mg/dL

Calcium

9.1 mg/dL

Albumin

4.1 g/dL

Which of the following is the most appropriate diagnostic study at this time?

2.

A 60-year-old man comes to the clinic for evaluation of a worsening cough. He has a long history of a chronic cough that has bothered him for at least 3 years. The cough is productive of thick white or yellow sputum and occurs almost daily. Over the last few days, however, he has had increased sputum production that is green tinged and low-grade fevers. He has a long history of tobacco use, having smoked at least a pack a day for the last 40 years. Physical examination reveals a large, mildly cyanotic appearing man in mild distress. He displays pursed lip breathing and is using accessory muscles of respiration with each breath. Chest examination reveals a prolonged expiratory time, coarse upper respiratory breath sounds, and a mild wheeze on expiration. A review of a recent set of pulmonary function tests, which include spirometry, lung volume, and diffusion capacity measurements, reveals a markedly diminished FEV1 (forced expiratory volume in 1 second) of approximately 1 L. Which of the following is an additional finding likely to be seen on pulmonary function testing? (A) (B) (C) (D) (E)

Decrease in the functional reserve capacity Equivalent decrease in forced vital capacity Increase in diffusion lung capacity of CO Increase in total lung capacity and residual volume Short and narrow flow-volume tracings

(A) Abdominal CT scan with contrast (B) Low-dose dexamethasone suppression test (C) Plasma renin activity and plasma aldosterone concentration (D) Renal biopsy (E) Twenty-four hour urine cortisol

1

USMLE Step 2 Assessment Exam

3.

A 30-year-old woman comes to the clinic complaining of an annoying cough that she believes she caught from her daughter. Apparently, her 3-year-old daughter started attending a new day-care center earlier in the week and quickly developed a productive cough. The patient reports suffering 2 days of cough productive of thick yellow sputum, low-grade fever, sore throat, and malaise. Before this episode, she has been healthy. She does not smoke, and, aside from the birth of her daughter, she has not spent a day in the hospital. Vital signs are unremarkable; the patient is afebrile and denies any fevers or chills at home. Cardiac examination reveals a regular rate and rhythm. Chest auscultation is remarkable for coarse upper-respiratory breath sounds, but no crackles, egophony, or dullness to percussion. She is breathing comfortably, without use of accessory muscles of respiration, at a rate of 14/min. There is no lymphadenopathy, pharyngeal exudate, sinus pain, bronchospasm, or dyspnea. Given this patient’s likely condition, which of the following is the most appropriate next step in her management? (A) (B) (C) (D) (E)

2

Chest radiograph Empiric macrolide antibiotics No treatment; follow clinically Sputum culture and Gram stain Viral serologies and culture

4.

A 49-year-old woman comes to your clinic complaining of new-onset rotational vertigo. It began approximately 2 months before and occurs several times per day without obvious provoking factors. She describes the sensation as “things flying around in circles,” requiring her to sit down or lean against something for balance. She has nausea with a few episodes of vomiting that accompany these dizzy spells and has had increased problems with hearing lately. She denies any other changes in her vision, cognition, memory, motor strength, or sensation. She has a past medical history of highly symptomatic menopause, anxiety, and uterine fibroids. She takes paroxetine, 50 mg once/day, and estrogen replacement therapy. A Hallpike test causes no change in her pattern of dizziness. Her hearing is slightly diminished bilaterally but worse on the left. The rest of her neurologic examination is normal. Over the course of the next several years, this patient most likely will experience which of the following? (A) Complete loss of her vestibular sensation of balance (B) Complete resolution of her symptoms (C) A continued increase in the frequency and severity of her vertigo symptoms (D) Development of refractory tinnitus (E) Resolution of her vertigo but continued hearing loss

Internal Medicine

5.

A 22-year-old African American woman comes to the clinic complaining of fatigue and a rash. Her symptoms have been intermittent and progressive over the last 2 years, finally prompting her to seek care. In addition to her rash, which mainly involves the malar portions of her face and occasionally the area around her ears, she has suffered from brittle hair, patchy alopecia, diffuse arthralgias and myalgias, and intermittent pleuritic chest pain. Her mood has become more labile over the year, and she feels irritable for no apparent reason. Other than these symptoms, she has been well and has not suffered any major illnesses. She currently takes no medications. Physical examination confirms the suspect butterfly rash and reveals two small oral ulcers that the patient had not noticed. Laboratory studies are remarkable for a normocytic anemia and a borderline elevated creatinine. Given this patient’s likely diagnosis, which of the following is the most specific autoantibody used to diagnose her disease? (A) (B) (C) (D) (E)

Anticentromere antibodies Antihistone antibody Antinuclear antibody Anti-Scl 70 antibody Anti-Sm antibody

6.

A 39-year-old woman comes to the emergency department complaining of the sudden onset of blanching and pain of her right third digit. It has occurred episodically over the past 3 months and is worsening in severity. Episodes come on suddenly, especially during times of emotional stress and exposure to cold. The blanching lasts for approximately 30 minutes and is followed by pain in the affected digit, then cyanosis, then flushing. Between episodes she has no complaints, and none of her other fingers have been affected. She denies any weight loss, sleep disturbance, fevers, chills, night sweats, or rashes. She takes oral contraceptive pills, naproxen for low back pain, and occasional sumatriptan for migraine headaches. Her past medical history is remarkable only for excessive hemorrhage following the birth of her third child. There is no other history of a clotting or bleeding disorder. She is in mild distress, but physical examination is normal except for her right third digit. It is markedly cyanotic with a sharp demarcation at the base of the finger. There are no skin changes, and it is nontender to palpation. Routine laboratory studies, including a complete blood count, electrolytes, creatinine, and urinalysis, are all normal. Which of the following medications is most likely to be effective in controlling this patient’s symptoms? (A) (B) (C) (D) (E)

Amitriptyline Gabapentin Nifedipine Prazosin Verapamil

3

USMLE Step 2 Assessment Exam

7.

A 43-year-old man comes to the clinic with his wife to discuss a worsening illness. Over the last year, the patient has suffered progressive neuropsychiatric symptoms, including severe depression and hallucinations. Additionally, the patient’s wife notes that he has become increasingly bizarre and labile and often seems unable to control his emotions. The patient was referred for cognitive testing, which revealed impairment mainly in psychomotor speed, attention, and concentration, with fairly intact verbal learning and memory. Additionally, over the past few months, the patient has developed strange jerking motions of his arms that are involuntary and occur without warning. Before this last year, the patient had no significant medical history. When asked about his family medical history, the patient is uncertain as to how his father died, though he knows his father suffered from a severe disease for the last 10 years of his life. Which of the following is the most likely underlying cause of this patient’s condition? (A) Abnormal copper regulation, resulting in cerebral copper deposition (B) Diffuse dopaminergic neuronal loss in the substantia nigra (C) Elevated total body iron stores, resulting in diffuse iron deposition (D) Extracellular deposition of beta-amyloid and neurofibrillary tangles (E) Trinucleotide repeats, resulting in neurotoxic protein expression

4

8.

A 62-year-old alcoholic man is brought to the emergency department complaining of nausea. He is bright yellow and obviously drunk, as he smells of alcohol. The patient is now complaining of severe nausea and dull but mild right upper quadrant pain that has persisted for the last 2 days. He reports having his last drink, a fifth of whiskey, earlier in the morning. Vital signs are: temperature 38.0 C (100.4 F), blood pressure 100/42 mm Hg, pulse 98/min, and respirations 22/min. Physical examination reveals a thin, jaundiced man in mild distress. Although lung and cardiac examination are unremarkable, abdominal examination reveals a tender, palpable liver 10 cm below the costal margin. There are scattered angiomata across the patient’s chest. Laboratory studies, including a full set of liver function tests, a complete blood count, a coagulation panel, and serum chemistries, are drawn and currently pending. The patient is planned for admission with a presumptive diagnosis of alcoholic hepatitis. If this patient is indeed suffering from alcoholic hepatitis, which of the following is an expected pattern of liver function tests? (A) AST level that is less than the ALT level, with both greater than 1000 U/L (B) AST more than twice the ALT level, ALT level less than 500 U/L (C) Extremely high aminotransferase levels, greater than 2000 to 3000 U/L (D) Near normal alkaline phosphatase and gammaglutamyltransferase levels (E) Total protein level that is more than two to three times the albumin level

Internal Medicine

9.

A 42-year-old woman comes to the clinic requesting analgesics for pain that began suddenly the day before. She began to feel nauseated and had two episodes of emesis shortly after the pain began. She describes the pain as wave-like, confined to the right side, and originating in the back. She also notes pain in the groin. She denies ever having had pain like this before. Her past medical history is remarkable only for three normal pregnancies, and her only medications are calcium supplements. Her temperature is 37.0 C (98.6 F), blood pressure is 145/80 mm Hg, pulse is 108/min, and she seems acutely distressed. Head, neck, lung, and cardiac examinations are normal. Abdominal examination is unremarkable and free of tenderness. There is marked tenderness on percussion of the costovertebral angle. Urinalysis shows 2+ blood but is otherwise negative. Laboratory studies show: Sodium

141 mEq/L

Potassium

4.4 mEq/L

Chloride

108 mEq/L

Bicarbonate

23 mEq/L

Blood urea nitrogen

14 mg/dL

Creatinine

0.8 mg/dL

Calcium

11.9 mg/dL

Albumin

3.8 g/dL

10.

(A) (B) (C) (D) (E)

11. Further questioning and examination would most likely reveal which of the following? (A) (B) (C) (D)

Chronic diarrhea Exertional dyspnea Swollen, painful joints in the fingers and toes Tetany in arm muscles when blood pressure measurement is attempted (E) Vague periumbilical abdominal pain

A 48-year-old man with a long history of diabetes mellitus type 2 is being treated for hyperosmolar nonketotic coma. Earlier in the day he was found confused and delirious, sleeping on a park bench. Paramedics brought the man to the hospital, where he was found to have a markedly elevated blood glucose level. The patient was started on aggressive fluid rehydration with normal saline and was started on an insulin drip. The patient was admitted to the medicine service for close observation and appropriate electrolyte replacement. In addition to the patient’s history of diabetes, he suffers from chronic hepatitis C, alcoholism, and early diabetic nephropathy. Currently, the patient is more alert and is continuing to receive intravenous fluids and insulin and is undergoing regular electrolyte checks. Shortly after the patient’s last laboratory draw, a worried nurse calls to let the resident know that the patient’s sodium level is 125 mEq/L and has been since he was transferred to the floor. To interpret this laboratory value, the resident should also ask the nurse for the measured level of which of the following? Albumin Alcohol Creatinine Glucose Potassium

A 49-year-old man has an episode of colicky left flank pain with radiation to the inner thigh and groin that lasts for approximately 3 hours and subsides spontaneously. He is advised by his physician to collect and filter all his urine, in the hope of identifying the urinary stone that he is suspected of having passed. He indeed recovers a 2-mm stone that on further study is determined to be a uric acid stone. An intravenous pyelogram shows a 3-mm filling defect present in the left renal pelvis, just above the ureteropelvic junction, and a CT scan without contrast shows a bright white spot of the same size on the same location. Further management should include which of the following? (A) Acidification of the urine and increased fluid intake (B) Endoscopic extraction of the stone (C) Long-term antibiotic therapy (D) Shock wave lithotripsy (E) Urinary alkalinization and increased fluid intake

5

USMLE Step 2 Assessment Exam

12.

A 39-year-old man with a history of ulcerative colitis has developed crampy right upper quadrant pain over the past year. The pain has been intermittent and seems somewhat related to meals. When asked specifically, he admits that recently he has been fatigued and incredibly itchy, though he has been taking an over-the-counter histamine blocker to help with the pruritus. He denies any recent weight loss, change in bowel habits, hematochezia, or melena. Examination reveals mildly icteric sclera and mild right upper quadrant tenderness. Laboratory studies reveal:

13.

Liver function tests Bilirubin, total

2.9 mg/dL

Alkaline phosphatase

259 U/L

AST

52 U/L

ALT

60 U/L

Additionally, a gamma-glutamyltransferase level is markedly elevated. Immunologic testing reveals an elevated antineutrophil cytoplasmic antibody (ANCA) titer. An abdominal ultrasound reveals some biliary sludge with moderate dilatation of the intra- and extrahepatic bile ducts. Given this patient’s likely condition, which of the following is the most likely cause of his recurrent symptoms? (A) (B) (C) (D) (E)

Biliary stricture Cholesterol gallstones Colon cancer Hepatoma Pigment gallstones

A 58-year-old man comes to the clinic complaining of abdominal and chest discomfort. He has had similar symptoms for many years that were always relieved by antacids or ranitidine as needed. They begin within 1 hour of eating and are exacerbated by spicy foods. Lately his symptoms have become steadily worse, and he awakens often from sleep with midepigastric burning and an acid taste in his mouth. He denies any weight loss, fevers, chills, or night sweats. His stools have been normal in color, without any blood. His past medical history is significant for hypertension, allergic rhinitis, and asthma. He takes atenolol and low-dose aspirin, loratadine, and a corticosteroid inhaler. He uses albuterol inhalers and ranitidine as needed. Physical examination is unremarkable, except for mild midepigastric tenderness. Which of the following is the most appropriate next step in this patient’s management? (A) Empiric therapy with a proton pump inhibitor such as pantoprazole (B) Esophagogastroduodenoscopy (EGD) (C) Fecal occult blood testing (D) Serology for Helicobacter pylori (E) Upper gastrointestinal series

14.

A 40-year-old man comes to the clinic complaining of fatigue. He also reports mild left upper abdominal and back pain, early satiety, and an unintentional 15-lb weight loss over the past few months. Before this, he has been healthy, never having spent a day in the hospital. His family history is unremarkable; the patient has a twin brother and an older sister, both of whom are healthy. Physical examination is remarkable for splenomegaly and mild tenderness over the lower sternum. A complete blood count shows: Leukocytes

85,000/mm3

Hematocrit

31%

Platelets

650,000/mm3

A bone marrow biopsy reveals granulocytic hyperplasia. Genetic analysis reveals a translocation between chromosomes 9 and 22. Which of the following is the most appropriate treatment for this patient’s condition? (A) (B) (C) (D) (E)

6

Cytarabine/busulfan Hydroxyurea Interferon-alpha Interferon-beta Stem cell transplant

Internal Medicine

15.

A 50-year-old man is seen in the emergency department complaining of abdominal pain and diarrhea. After evaluation and rehydration, the patient is discharged home with a diagnosis of viral gastroenteritis. During his workup, however, the intern taking care of the patient ordered a full array of liver function tests, which revealed an elevated gamma-glutamyl transpeptidase (GGT). The patient was advised to follow up with his primary care physician. The patient now presents in the clinic for evaluation of this isolated laboratory abnormality. He reports feeling healthy since recovering and never having a history of liver trouble. When asked, he reports drinking only socially and denies any illicit drug use. Current medications include acetaminophen for osteoarthritis, atorvastatin for dyslipidemia, amiodarone for a persistent supraventricular tachycardia, and metformin for recently diagnosed diabetes. In addition to these conditions, he suffers from depressive symptoms and was recently fired from his job. Physical examination is unremarkable except for mild rhinophyma. Which of the following is most likely to explain this patient’s elevated GGT level? (A) (B) (C) (D) (E)

Acetaminophen Alcohol Amiodarone Atorvastatin Metformin

16.

A concerned nurse pages the resident in the middle of the night about a patient who is 1 day post a coronary artery bypass graft procedure and who is currently in the surgical step-down unit. Apparently, the patient is confused, agitated, and having difficulty communicating. The nurse reports that his blood pressure is 95/42 mm Hg and pulse is 15/min, according to the cardiac monitor. He is breathing at 22/min and has a pulse oxygen saturation of 95%. Rushing to the patient’s room, the resident finds the patient calm, though somewhat somnolent, and lying in bed. He is able to communicate, albeit slowly, but denies chest pain, shortness of breath, or any other symptoms, and just wants to be left alone. The patient appears diaphoretic and in mild distress. The peripheral pulse is weak and thready, and his extremities are cool. The cardiac monitor displays the following rhythm:

The nurse asks if a crash cart is needed, and whether a code should be called. Which of the following is the most appropriate first intervention? (A) (B) (C) (D) (E)

Amiodarone bolus Electrical cardioversion Epinephrine, intravenous Lidocaine bolus and drip Transfer to cardiac care unit

7

USMLE Step 2 Assessment Exam

17.

A 52-year-old woman is being evaluated for fatigue and an abnormal gait. She has noticed that over the last year she has become increasingly likely to trip and often has difficulty maintaining her balance. Further, she feels exhausted most of the day and tires with minimal exertion. She is increasingly forgetful and often has episodes where she feels confused and cannot think clearly. Also, her hands and feet occasionally burn and tingle. Physical examination is remarkable for a smooth tongue with mild cheilosis, hyperactive reflexes, and a wide-based gait. Laboratory studies reveal a severe macrocytic anemia with a less severe neutropenia and thrombocytopenia. A review of the peripheral blood smear shows few reticulocytes and numerous hypersegmented neutrophils. Which of the following is the most appropriate next step to definitively diagnose this patient’s condition? (A) (B) (C) (D) (E)

Bone marrow biopsy Ham test Lumbar puncture MRI of brain/spine Schilling test

18.

A 58-year-old man comes to the clinic complaining of shortness of breath. He described the dyspnea as gradual in onset over the last few weeks. The patient denies any pleuritic pain, cough, or orthopnea, and has no previous cardiac or respiratory medical history. Review of systems, however, is concerning for an unexplained 20-lb weight loss over the last few months and intermittent fevers and night sweats. The patient does not smoke cigarettes currently, though he has a distant 20 pack-year history. He denies using illicit substances and only rarely drinks alcohol. He has not had any major illnesses in the past, has had no major operations, and is employed as an accountant. Physical examination is remarkable for reduced breath sounds and dullness to percussion approximately halfway up the right hemithorax. Chest radiograph confirms a right-sided pleural effusion and some hilar and mediastinal lymphadenopathy. A diagnostic thoracentesis is performed that reveals a milky-white, opalescent fluid. Laboratory studies reveal this fluid to have a markedly elevated triglyceride level of 558 mg/dL, a high cell count composed mainly of T lymphocytes, and numerous chylomicrons. Cytologic examination is unremarkable. Which of the following is the most likely cause of this patient’s effusion? (A) (B) (C) (D) (E)

8

Infection with acid-fast organisms Lymphangioleiomyomatosis (LAM) Non-Hodgkin lymphoma Pancreatic adenocarcinoma Undiagnosed mesothelioma

Internal Medicine

19.

A 24-year-old man comes to the clinic complaining of a painful neck. For the past week, he has had an exquisitely tender lump in his neck. Additionally, he has suffered from heart palpitations and insomnia, and he reports feeling “grouchy.” Although his past medical history is unremarkable, a review of systems reveals a recent history of cough, fever, and myalgias, now resolved. Vital signs are: temperature 38.0 C (100.4 F), blood pressure 138/88 mm Hg, pulse 121/min, and respirations 20/min. Examination reveals a palpable, non-nodular, and extremely tender thyroid gland. A mild tremor is present when the patient extends his arms. Thyroid function tests show: Thyroid-stimulating hormone (TSH)

0.01 µU/mL

Thyroxine (T4)

12.8 µg/dL

A 24-hour radiouptake study performed on this patient most likely would reveal which of the following? (A) Diminished radioactive iodine uptake throughout the entire thyroid gland (B) Focal area of increased uptake; rest of the gland displaying reduced uptake (C) Increased uptake of radioactive iodine throughout the entire thyroid gland (D) Nodular areas of increased uptake surrounded by areas of reduced uptake (E) Normal radioactive iodine uptake scan; no abnormalities noted

20.

A 22-year-old woman comes to the clinic complaining of severe abdominal pain. About 10 days ago, shortly after returning from a 2-week vacation in Mexico, she developed fevers and diffuse, crampy, abdominal pain. Additionally, she reports nausea, malaise, and dark urine and that friends have commented more on her yellow-tinged eyes than her dark tan. Given these symptoms and recent travel history, as well as a marked transaminitis, the patient is evaluated for a possible viral hepatitis. A hepatitis screening panel shows: IgM anti-HAV

Positive

IgG anti-HAV

Negative

Hep B surface Ag

Negative

Hep B surface Ab

Positive

IgG anti-Hep C

Negative

The patient is treated symptomatically, and symptomatically improves over the course of a month. By 3 months, her liver enzymes have normalized. Which of the following is an additional step in the management of this patient? (A) (B) (C) (D) (E)

Hepatitis A vaccination Hepatitis B vaccination Liver biopsy for evaluation of long-term sequela Reporting the case to public health officials Right upper quadrant ultrasound and alphafetoprotein level

9

USMLE Step 2 Assessment Exam

21.

A 58-year-old woman comes to the emergency department complaining of chest pain. The pain is substernal, does not radiate, and is worse with inspiration and recumbency. She is not sure when the pain started but thinks it may have been occurring intermittently for about 6 hours. Her past medical history is significant for diabetes mellitus type 2, hypertension, and diabetic nephropathy. Surgical history includes an appendectomy as a child and a recently created atriovenous fistula in the right arm in anticipation of dialysis. She denies tobacco or alcohol use. Her pulse is 122/min. Multiple “squeaky” sounds are heard on cardiac auscultation. Additionally, the patient has 2+ pitting edema bilaterally. An electrocardiogram reveals only sinus tachycardia. Laboratory studies show moderate normocytic anemia, a mildly increased troponin T level, mildly elevated potassium, and significantly elevated BUN and creatinine levels. Which of the following is the most appropriate treatment for this patient’s chest pain? (A) (B) (C) (D) (E)

Calcium gluconate Cardiac catheterization Hemodialysis Heparin drip Indomethacin

22.

A 40-year-old woman comes to the emergency department complaining of substernal chest pain. The patient reports having intermittent, nonradiating, dull substernal chest pain and pressure that can last up to half an hour. Today, however, her pain seemed worse than usual, prompting her to come to the emergency department. Although she denies any shortness of breath, diaphoresis, nausea, or vomiting, she does note that she has difficulty swallowing during these episodes of chest pain. Her past medical history is significant for stage one hypertension, currently untreated, and mild gastroesophageal reflux disease (GERD) diagnosed 1 year ago for which she is treated with a proton pump inhibitor. The patient’s chest pain quickly resolves with two sublingual nitroglycerin tablets and she is admitted for cardiac evaluation and monitoring. Over the next 24 hours, she exhibits no electrocardiographic changes or increases in her cardiac enzymes, though she has some recurrent chest pain, again relieved with nitroglycerin and a “GI cocktail.” The next morning, the patient has a normal cardiac thallium perfusion scan and is discharged with the diagnosis of “atypical chest pain.” Which of the following is an appropriate next step in the outpatient evaluation of this patient? (A) (B) (C) (D)

Cardiac catheterization and angiography Esophageal pH probe and manometry Transthoracic echocardiogram Trial of H2 blocking agent

(E) Upper endoscopy with gastric biopsy

10

Internal Medicine

23.

A 45-year-old woman is recovering from transsphenoidal removal of a pituitary adenoma. Although the surgery apparently went well and there were no intraoperative complications, the patient’s postoperative course has been complicated by polyuria, with the patient losing more than 5 liters of urine a day, currently being replaced with D5 water. Additionally, the patient’s sodium has been increasing slowly. The internal medicine resident is consulted. A water restriction test followed by the administration of intramuscular antidiuretic hormone is performed, and it results in an increase in urine osmolality. Recent laboratory results are as follows: Pre-Op

Post-Op Day 3

Plasma sodium

138 mEq/L

159 mEq/L

Urine osmolality

800 mOsm/Kg

123 mOsm/Kg

Should this patient’s sodium continue to increase, an appropriate intervention would be to administer which of the following? (A) (B) (C) (D) (E)

Demeclocycline Desmopressin Furosemide Hypertonic saline Normal saline

24.

A 32-year-old woman comes to the clinic complaining of malaise, severe pallor, and generalized weakness. Approximately 2 months ago, while working at a rural health clinic in the war-torn Congo, she became extremely ill with a severe gastroenteritis. She was given antibiotics, though she is not sure which ones, and quickly recovered. Since returning to the United States approximately 6 weeks ago, however, she has felt progressively more ill. She has never been sick in the past, takes no medications, and knows of no diseases that run in her family. A review of symptoms, aside from reduced exercise tolerance and a mild decrease in appetite, is unremarkable. She has suffered no recent fevers, chills, nausea, or diarrhea, and denies any weight loss or night sweats. Vital signs are: temperature 37.0 C (98.6 F), blood pressure 118/70 mm Hg, pulse 103/min, and respirations 20/min. Examination reveals a young, pale woman in no apparent distress. Cardiac auscultation reveals a 2/6 midsystolic murmur heard best at the right upper sternal border that does not radiate. Lungs are clear to auscultation bilaterally, and abdominal examination is benign, without evidence of hepatosplenomegaly. There are no signs and stigmata of liver disease or endocarditis. Laboratory studies show: Hematologic Hematocrit

17%

Hemoglobin

6.0 g/dL

Leukocytes

3000/mm3 (67% neutrophils, 25% lymphocytes, 7% monocytes)

Platelets

12,500/mm3

Reticulocytes

0.6%

The rest of the patient’s laboratory studies, including a set of chemistries and liver function tests, are unremarkable. Which of the following is the next most appropriate diagnostic study? (A) (B) (C) (D) (E)

Blood and stool cultures Bone marrow biopsy CT scan of abdomen Echocardiogram Erythropoietin level

11

USMLE Step 2 Assessment Exam

25.

A 28-year-old woman is being worked up for recurrent venous clots in her hepatic circulation. Her diagnosis has eluded many physicians, as her symptoms are confusing and difficult to relate. In addition to these clots, she has a seemingly intermittent anemia and occasionally full pancytopenia, though she always has a reticulocytosis. Her anemia has been severe enough to warrant transfusion in the past. Shortly after an appropriately matched transfusion, however, she suffered shaking chills and hemoglobinuria. Additionally, she also has been documented to have intermittent hemoglobinuria and reports occasionally seeing blood in her urine in the morning. An extensive hypercoagulability workup has been unrevealing. Multiple reviews of the patient’s peripheral blood smear have failed to reveal any morphologic abnormalities, and a bone marrow smear, aside from increased cellularity, is unrevealing. Iron levels are low, as is serum haptoglobin. Given the patient’s intermittent symptoms and evidence of hemolysis, there is concern that there is a defect in the cell membrane stability of the peripheral blood cells. Which of the following is an appropriate diagnostic test to order to evaluate this hypothesis? (A) (B) (C) (D) (E)

12

Complement levels Gel electrophoresis Ham acidification test Lactate dehydrogenase level Osmotic fragility test

26.

An 18-year-old man comes to clinic complaining of fatigue and pallor. He states that he has suffered from episodic anemia for as long as he can remember, and now that he can make his own medical decisions he would like to have it treated before he leaves for college. Aside from this recurrent anemia, he has had no major illnesses. The only medication he takes is a daily multivitamin. Physical examination confirms a mild pallor and marked splenomegaly. Laboratory studies are remarkable for a reduced hematocrit with an elevated mean corpuscular hemoglobin concentration (MCHC) and elevated red cell distribution width (RDW). A review of the blood smear shows numerous small, dense, hyperchromatic erythrocytes that lack central pallor. A review of the patient’s previous laboratory studies reveals a positive osmotic fragility test, a low haptoglobin level, and an elevated lactate dehydrogenase level. Which of the following is the most appropriate treatment for this patient’s anemia? (A) Corticosteroids (B) Erythropoietin (C) No treatment (D) Splenectomy (E) Stem cell transplant

Internal Medicine

27.

A 70-year-old man comes to the emergency department complaining of chest palpitations and light-headedness. These symptoms have occurred intermittently for years but now seem to be occurring almost daily. The symptoms usually come and go in a matter of minutes. This morning, however, the symptoms did not resolve, prompting the patient to seek medical care. He has a blood pressure of 132/88 mm Hg, pulse of 170/min, respirations of 22/min, and a temperature of 37 C (98.6 F). Examination is remarkable for runs of a normal rhythm interspersed with a regular tachycardia. A review of the patient’s rhythm strip reveals an alternating normal sinus rhythm and a narrow complex tachycardia. Before the patient’s runs of tachycardia, an atrial premature beat can be seen. Immediately following the Q wave is a small upward deflection on the rhythm strip, which the physician refers to as “a pseudo-R wave.” On the basis of this information, the patient receives a carotid massage and is instructed on how to perform a Valsalva maneuver. Despite these interventions, his tachycardia continues. Which of the following is the next appropriate intervention to treat this patient’s tachycardia? (A) (B) (C) (D) (E)

28.

Adenosine Amiodarone Amlodipine DC cardioversion Digoxin

A 69-year-old woman with a long-standing history of hypertension reports progressive dyspnea on exertion over the past few weeks to months, as well as not being able to walk as much as she used to due to “tiring.” She sleeps with three pillows and often wakens at night feeling very short of breath. She has smoked a pack of cigarettes a days for the past 40 years. She has never been hospitalized. Cardiopulmonary examination reveals bibasilar rales, an S4, and a weak apical impulse in the sixth intercostal space. There is 2+ pitting edema of her lower extremities. Which of the following is the most appropriate next step in management? (A) Admit her to the hospital for an echocardiogram and further treatment (B) Instruct her to limit her physical activity (C) Instruct her to limit her salt intake (D) Prescribe a diuretic and re-evaluate in 3 months (E) Send her for an echocardiogram and re-evaluate in 3 months

29.

A 33-year-old man is brought to the emergency department with altered mental status. The man was found during a security check of a large transatlantic cargo ship shortly after the boat docked in the United States. Apparently, he is a stowaway who somehow boarded the ship while it was docked in West Africa to deliver relief supplies. He is obviously confused and disoriented but is able to report boarding the ship to escape from extreme malnourishment in his country. Currently, he is complaining of diarrhea and nausea. Physical exam reveals an anxious, mildly disoriented man with a large, symmetric, hyperpigmented rash on his forearms. He has significant temporal wasting and a beefy red tongue. The rest of the examination is unrevealing. Full laboratory work is still pending but a complete blood count is available and is remarkable only for a mild normocytic anemia. Given this patient’s condition, in addition to caloric support, which of the following is the most appropriate supplement to treat his symptoms? (A) (B) (C) (D) (E)

30.

Niacin Thiamine Vitamin B6 Vitamin B12 Vitamin C

A pregnant 28-year-old woman comes to the same-day clinic one morning in late October requesting an influenza vaccination. She recently found out she needs to be immunized annually for work because the nursing home where she is employed requires it of all employees. Although she has had the vaccination in the past with no complications, she is concerned that she could harm her second-trimester fetus. She has no significant past medical history or comorbidities and has never had a particularly severe case of the flu. Her pregnancy has so far been without complications aside from some mild morning sickness. Which of the following is the most appropriate immunization strategy in this pregnant patient? (A) (B) (C) (D) (E)

Give the influenza immunization during this visit No immunization; prescribe as-needed oseltamivir Provide prophylactic amantadine instead of vaccine Provide vaccination, but discourage breast-feeding Recommend immunization immediately postpartum

13

USMLE Step 2 Assessment Exam

31.

A 24-year-old man comes to the clinic complaining of flu-like symptoms. For the past 10 days, he has had diffuse myalgias, low-grade fevers, headache, and occasional nausea and vomiting. He reports being generally healthy in the past, though a review of his chart reveals that he was treated for gonorrhea 4 weeks ago. Exam is remarkable for a diffuse, tender lymphadenopathy as well as two small, painful mucocutaneous ulcers in his oropharynx. On further discussion with the patient, he reveals that he has numerous HIV risk factors, including intravenous drug use and sex with commercial sex workers. Which of the following is the most appropriate HIV test for this patient? (A) (B) (C) (D) (E)

32.

14

CD4+ T-cell count ELISA for HIV antibody HIV RNA level P24 antigen serology Western blot

A 60-year-old man is being treated for an elevated lowdensity lipoprotein (LDL) level. Before being started on an HMG CoA reductase inhibitor, he receives a baseline set of liver function tests, which show a markedly elevated alkaline phosphatase level. The patient denies feeling ill and takes no current medications. His past medical history, aside from dyslipidemia, is remarkable only for a humeral fracture sustained while skiing and for accelerated presbyacusis. A full review of systems and physical examination are unremarkable. In trying to differentiate the source of this patient’s elevated alkaline phosphatase, which of the following is the most appropriate next test to order? (A) (B) (C) (D) (E)

33.

Alanine aminotransferase Aspartate aminotransferase Gamma-glutamyl transpeptidase Lactate dehydrogenase isoenzymes Repeat alkaline phosphatase

Paramedics rush a 48-year-old man who has suffered a witnessed seizure to the emergency room. The patient was found in the park, lying on the ground convulsing and drooling, at which time emergency medical help was called. He stopped having tonic-clonic movements while being transferred to the ambulance. However, during transit he had repeated generalized seizure activity, which is currently ongoing. In the emergency room, the patient is unresponsive and is having repetitive movements of his arms, neck, and torso. He is coughing up blood and appears to have bitten his tongue. According to the paramedics’ estimates, the patient has had some degree of seizure activity for at least the last 30 minutes. Which of the following is the most appropriate first step in his management? (A) (B) (C) (D) (E)

34.

Diazepam, liquid EEG monitor placement Intubate the airway Phenobarbital infusion Rectal diazepam

A 58-year-old man comes to the emergency room with crushing substernal chest pain. After an evaluation in the emergency room, he is admitted to the cardiac care unit with an acute inferior myocardial infarction. A few hours after admission, his blood pressure is 90/52 mm Hg and pulse is 42/min with a normal sinus rhythm. The patient is normally hypertensive and had an admission pulse of 110/min. He is currently pain-free, though he feels a bit light-headed and slow to respond to questions. Which of the following is the most appropriate management? (A) (B) (C) (D) (E)

Emergency CABG Intravenous atropine Phenylephrine drip Transcutaneous pacing Transvenous pacing

Internal Medicine

35.

A 28-year-old African American man comes to the emergency department complaining of urinary tract symptoms. He is given oral trimethoprim/sulfamethoxazole and told to follow up with his primary care physician. Three days later, he comes to clinic complaining of fatigue, shaking chills, and dark-colored urine. He has had a similar episode once before, as a child, and was told he had a food allergy to beans. Physical examination, aside from some mild pallor, is unremarkable. Laboratory studies show:

36.

Hematologic Hemoglobin

7.5 g/dL

Hematocrit

24%

Leukocytes

4000/mm3

Platelets

143,000/mm3

Reticulocytes

7%

(A) (B) (C) (D) (E)

A peripheral blood smear is remarkable for polychromasia and numerous acanthocytes. Which of the following is the most likely underlying cause of this patient’s condition? (A) (B) (C) (D) (E)

Abnormal cellular membrane proteins Drug-induced autoantibody formation Erythrocyte enzymatic deficiency Hapten-mediated cellular destruction Inherited hemoglobinopathy

A 55-year-old man with a long smoking history comes to the clinic for evaluation of a chronic cough. The cough has bothered him for at least 3 years now. It is productive of thick white or yellow sputum and occurs on most days. He sheepishly admits that he has a long history of tobacco use, having smoked at least a pack a day for the last 40 years. Physical examination reveals some mild wheezes on expiration and coarse upper respiratory breath sounds. Pulse oximetry is remarkable for an oxygen saturation of 93%, which drops to 91% with exercise. Pulmonary function tests reveal a decreased forced expiratory volume in 1 second with a relatively preserved forced vital capacity, increased lung volumes, and a moderately reduced diffusion lung capacity of CO. In addition to smoking cessation, which of the following is the most effective and appropriate long-term outpatient medical treatment for this patient?

37.

Albuterol Flunisolide Ipratropium Oxygen therapy Prednisone

A 25-year-old woman comes to the clinic complaining of pain and swelling in the right elbow. She noticed the swelling 2 days before presentation. She reports that she has been feeling somewhat feverish for the last several days and also reports nonspecific symptoms such as arthritis and myalgia. She denies any other medical history. She is not taking any medications. She denies any drug use. Her temperature is 38.3 C (101.0 F), blood pressure is 120/80 mm Hg, and pulse is 100/min. She has a vesiculopustular skin lesion on her abdomen and a mildly tender and swollen right elbow. Which of the following is appropriate pharmacotherapy? (A) (B) (C) (D) (E)

Acyclovir Azithromycin Ceftriaxone Ciprofloxacin Vancomycin

15

USMLE Step 2 Assessment Exam

38.

A 42-year-old woman received a lung transplant 5 months ago. She is on several immunosuppressants for her transplant. She now complains of several days of fever, chills, shakes, cough, and hemoptysis. She reports no recent long-distance travel for immobilization. She has no leg swelling or palpitation. A left upper lobe cavitary lesion is seen on chest x-ray. Induced sputum is obtained and returns positive for acid-fast bacilli. Laboratory studies are otherwise unremarkable. Mycobacterium tuberculosis infection is suspected. Which of the following is the most appropriate management of this patient? (A) (B) (C) (D) (E)

39.

Isoniazid Isoniazid and rifampin Isoniazid, rifampin, and pyrazinamide Isoniazid, rifampin, pyrazinamide, and ethambutol Rifampin

A 63-year-old man is admitted to the hospital for management of pneumococcal pneumonia. He has been previously healthy until he developed fevers and a cough productive of brownish sputum. He is admitted for administration of intravenous antibiotics. Over the next few days, he is noted to be progressively lethargic and confused. Evaluation for meningitis, including a lumbar puncture, is negative. A CT scan of the head is negative. Chest x-ray reveals a persistent pneumonia in the right lower lobe. His serum sodium is 125 mEq/L. Blood cultures and remaining laboratory studies are normal. His jugular venous pressure is 9 cm H2O. Examination reveals crackles in the right lower lung field. He has no pedal edema. The patient has been eating fairly well and has not been noted to be aspirating. His fractional excretion of sodium (FeNa) is 0.5%. Which of the following is the most appropriate management? (A) (B) (C) (D) (E)

40.

Administer a bolus of normal saline Administer furosemide Administer salt tablets Expand antibiotic coverage Restrict fluid to 1 L of water daily

A 30-year-old man comes to the emergency department complaining of chest palpitations. For the last few hours he has been light-headed and feels as if his heart is “bursting out” of his chest. He has had some similar fleeting episodes of palpitations in the past, but because they have been transient, he has simply attributed them to stress and coffee and has not sought emergency medical treatment. When checking the radial pulse, the physician notes an irregular, rapid heart rate of approximately 130/min. The rest of the patient’s vital signs are stable. An electrocardiogram shows atrial fibrillation with a ventricular rate as high as 230/min. The QRS complexes appear widened. An old electrocardiogram obtained 1 year ago as part of a pre-employment physical examination is available. It reveals a short PR interval with a strange up-sloping of the R wave. Which of the following is the most appropriate treatment for this patient’s arrhythmia? (A) (B) (C) (D) (E)

41.

A 21-year-old college student comes to the clinic concerned about a new tremor and increased difficulty concentrating. He has a history of behavioral disorders since high school and a previous diagnosis of attention deficit hyperactivity disorder (ADHD), but recently things have become much worse. Over the past year he has developed a tremor in both arms and difficulty concentrating. He has felt increasingly distant from friends, who have found his behavior to be odd and different, though he himself has noticed no change, aside from perhaps some mild irritability. A review of symptoms reveals some mild dysphagia, episodic dysarthria, and evidence of mild depression. His past medical history, aside from the ADHD, is significant for autoimmune liver disease treated with steroids. Ocular examination is remarkable for a dark greenish-brown deposit surrounding the iris. Abdominal palpation reveals marked hepatosplenomegaly; neurologic exam shows a mild tremor, dysmetria, and an ataxic gait. Which of the following is an expected laboratory abnormality in this patient that can be used to diagnose his disease? (A) (B) (C) (D) (E)

16

Digoxin Emergent electrical cardioversion Intravenous diltiazem Metoprolol Procainamide

Elevated serum copper concentration Elevated urinary copper excretion Increased circulating serum ceruloplasmin Low amount of circulating free copper Reduced hepatic copper concentration

Internal Medicine

42.

A 66-year-old obese woman has had deep, aching pain in her hands and right knee for the past 4 years. The pain occurs with use of the joint and is relieved with rest and cessation of weight bearing. She has no other symptoms and does not take any medications. Her temperature is 37 C (98.6 F). Physical examination shows joint margin tenderness and fine crepitations and limitations to joint motion. There is bony swelling of the proximal and distal interphalangeal joints. The remainder of the examination is unremarkable. In addition to weight loss and exercise, which of the following is the most appropriate initial treatment? (A) (B) (C) (D) (E)

43.

Acetaminophen Joint arthroplasty Methotrexate Prednisone No additional therapy is indicated

A 58-year-old man comes to the clinic for a health care checkup. He wishes to discuss his risk for having a myocardial infarction in the next 10 years, as his brother, who is 48 years old, recently suffered from a near fatal one. He denies any chest pain, shortness of breath, orthopnea, or other cardiac symptoms, though he admits he has been ill in the past. Past medical history is remarkable for diabetes mellitus, diagnosed 8 years ago, and a stroke 2 years ago. He recently quit smoking after a 40 pack-year history and is mildly overweight. Physical examination is unremarkable, though an electrocardiogram shows high voltages in the precordial leads. Given this patient’s risk factors, which of the following is the most appropriate strategy for cardiac risk stratification and to determine treatment goals? (A) (B) (C) (D) (E)

Coronary computed tomography Electrocardiographic exercise testing Exercise-stress echocardiography Myocardial perfusion scintigraphy No further testing necessary

44.

A 68-year-old man is recovering from a case of community-acquired pneumonia. He now returns to the clinic complaining of continued malaise, fevers, and chills, though his cough has improved somewhat. He has finished a 10-day course of levofloxacin, as prescribed, and feels it has helped him somewhat, though he is still feeling ill. Physical examination is remarkable for a low-grade fever and for reduced lung sounds and dullness to percussion one third of the way up the right hemithorax. A chest radiograph confirms the suspected effusion, and a diagnostic thoracentesis is performed, revealing a thick, exudative-appearing fluid. The effusion occupies approximately one third of the right hemithorax and partially layers on right lateral decubitus radiographs. There is some difficulty draining the fluid, but approximately 50 mL of the effusion is removed. Laboratory studies show: Pleural fluid Glucose

15 mg/dL

pH

7.04

Cell count

10,000 cells/mL, mainly PMNs

Gram stain

Numerous gram-positive cocci

Given this patient’s presentation, which of the following is the most appropriate management? (A) Admit for broad-spectrum antibiotics pending culture results (B) Admit patient to hospital for chest tube drainage and antibiotics (C) Prescribe oral ciprofloxacin; follow serial radiographs (D) Refer patient to cardiothoracic surgery for open pleurectomy (E) Repeat radiograph and perform repeat thoracentesis as necessary

17

USMLE Step 2 Assessment Exam

45.

A 34-year-old man with a known diagnosis of HIV/AIDS is brought to the emergency department following a witnessed seizure. Although he is still mildly confused in the emergency department, he reports a history of weakness, fatigue, chronic diarrhea, and, over the last week, progressive “heaviness” and weakness of the left side of his body. Reviewing his chart, the physician sees that his last CD4+ lymphocyte count was 148 cells/mm3, and the HIV viral load was 400,000 copies/mm3. Physical examination reveals a thin, cachectic-appearing man with a mild left hemiparesis. MRI reveals three ring-enhancing lesions, located in the right basal ganglia, the right frontal lobe, and the left parietal lobe. Which of the following is the most appropriate first treatment for this patient? (A) (B) (C) (D) (E)

Amphotericin B Glucocorticoids Pyrimethamine/sulfadiazine/leucovorin Surgical resection Trimethoprim/sulfamethoxazole

46.

A 58-year-old woman comes to the clinic for a routine medical checkup. She has no major medical problems and denies any recent illness aside from a couple of “colds” and an annoying, persistent cough. She does not have a history of asthma and has never had pneumonia. On review of systems, the patient reports feeling more fatigued, which she attributes to aging, though she denies any weight loss or other constitutional symptoms. She does not exercise regularly and has smoked one to one and a half packs of cigarettes per day since age 20 years. Her cough has bothered her for the last few years and seems to be getting worse. She reports producing approximately 1-2 tablespoons of white or yellow tinged sputum. Physical examination reveals a prolonged expiratory phase of respiration, scattered rhonchi that clear with coughing, and a faint end-expiratory wheeze. Spirometry shows: Measurement

Percent of Predicted

Forced vital capacity (FVC)

92%

Forced expiratory volume in 1 second (FEV1)

42%

Ratio of FEV1 to FVC

48%

Forced expiratory flow, midexpiratory (FEF25-75%)

28%

Given this patient’s presentation, which of the following is most likely to improve her condition over the next few years? (A) (B) (C) (D) (E)

18

Aggressive use of inhaled beta-agonists Daily use of inhaled glucocorticoids Intensive efforts aimed at smoking cessation Lung reduction surgery or transplantation Regular use of inhaled cholinergic antagonists

Internal Medicine

47.

A 33-year-old woman comes to the clinic complaining of fluctuating weakness and fatigability. She often gets tired walking up stairs or has difficulty rising from a chair. Additionally, she occasionally suffers from double vision and recently has begun to have episodes of slurred speech and difficulty speaking. Her symptoms usually improve with rest or sleep. These problems have plagued her for the last 2 years, but recently these “spells” seem to have been longer and more severe, though she currently feels “okay.” Review of systems is unremarkable; she denies any chest pain, orthopnea, dyspnea, muscle pain, seizure-like activity, weight changes, or other constitutional symptoms. Family and social history are likewise unremarkable; there are no neurologic diseases in the family and the patient has never smoked cigarettes. Physical examination reveals mild, bilateral ptosis, weakness of upward gaze, and mild proximal muscle weakness. With progressive testing, the patient’s upward gaze and hand strength become weaker. Based on the patient’s symptoms, a test dose of edrophonium is administered to elucidate the cause of her weakness. After receiving a total dose of 10 mg of edrophonium, which of the following is a likely response in this patient? (A) Gradual improvement in ptosis, diplopia, and dysarthria (B) No immediate effect from the first dose of edrophonium (C) Reproduction of severe symptoms of weakness (D) Temporary increase in pulse and sympathetic tone (E) Transient increase in the strength of her handgrip

48.

A 45-year-old man comes to clinic complaining of fatigue, palpitations, and dizziness made worse by standing. A focused history reveals that the patient has had recent episodes of melena, though he denies any hematochezia or hematemesis. A stat blood count confirms the suspected anemia, revealing a hematocrit of 25%. Although the patient does not have any clinical evidence of liver disease, he does have a 10-year history of hepatitis C. Although he has had normal liver function tests, a recent right upper quadrant ultrasound revealed an irregular, coarse-appearing liver and an enlarged spleen. Given this evidence of cirrhosis and portal hypertension, there is concern that the patient might have variceal bleeding. He is admitted for blood transfusion and an upper endoscopy. Endoscopy fails to reveal any evidence of esophageal or significant gastric varices. The gastric mucosa does, however, appear abnormal. In some areas a fine white reticular pattern is interspersed between pink, moist mucosa, creating a “snakeskin” appearance. Other areas, particularly in the body of the stomach, reveal increased vascularity, small areas of oozing, and multiple, punctate, subepithelial hemorrhages. The patient is given a tentative diagnosis of portal hypertensive (congestive) gastropathy as the cause of his bleeding. He is observed over the next 2 days and is found to have a stable hematocrit. Which of the following is an additional appropriate treatment for this patient? (A) Beta-blocker therapy (B) Histamine-H2 blocker (C) No additional treatment (D) Proton pump inhibitor (E) Surgical resection

19

USMLE Step 2 Assessment Exam

49.

A 28-year-old man with a history of hereditary spherocytosis comes to the clinic for a routine physical examination. He reports being in good health and has no current complaints, though he needs a health examination form filled out for insurance purposes. When asked about his history of hereditary spherocytosis, he reports having the disease as a child but having no problems since he was probably 8 years old, when he was treated. The patient currently takes no medications, and a review of systems is unremarkable. Physical examination reveals a well-healed surgical scar in the left upper quadrant but is otherwise unremarkable. Laboratory studies include a complete blood count with an elevated mean corpuscle hemoglobin concentration but a normal hemoglobin level. A review of the peripheral blood smear reveals numerous spherocytes and aggregates of denatured hemoglobin in the red blood cell periphery. Which of the following is an appropriate treatment for this patient? (A) (B) (C) (D) (E)

50.

An 84-year-old woman is brought from her assisted living facility to the urgent care clinic. She is forgetful and somewhat confused but is able to convey to you that she is having severe muscle cramps and generalized weakness. The symptoms have lasted at least a week, are worse in her lower extremities, and are unrelieved with rest. She is a poor historian but does state that she started a new medication for her high blood pressure 1 to 2 months ago. Unfortunately, she cannot remember the name of her high blood pressure medication, and the assisted living facility did not send a list of the patient’s medications. Assuming her current symptoms of weakness, leg cramps, and confusion are caused by her new medication, which of the following is the most likely culprit? (A) (B) (C) (D) (E)

20

Corticosteroids Dental antibiotic prophylaxis Folic acid supplements No additional treatment Pneumococcal vaccine

Captopril Clonidine Felodipine Hydrochlorothiazide Metoprolol

51.

A 68-year-old man comes to the clinic complaining of severe dyspnea on exertion. He also reports a chronic cough that is worse at night and that produces a small amount of whitish sputum. The symptoms have been progressive since his retirement 2 years ago from the family business, a large marble and quartz quarry. He denies any associated symptoms; he has had no hemoptysis, fevers, wheeze, or pleuritic pain. Past medical history is unremarkable, but the patient does admit to a 40 pack-year history of tobacco use. A review of systems is unrevealing. Physical examination is remarkable for dry inspiratory crackles bilaterally and clubbing of the fingers. A chest radiograph shows multiple 3- to 5-mm nodules, with a greater number of nodules in the upper lobes. Two larger, ill-defined masses are present in both the left and right upper lobes, though no obvious cavitation is seen. Pulmonary function testing reveals a severe ventilatory defect and a restrictive lung disease pattern. Shortly after this clinic visit, the patient dies from hypoxic respiratory failure. At the request of the family, an autopsy is performed. Given this patient’s presentation, which of the following is a likely finding on autopsy? (A) Evidence of chronic, postobstructive necrotizing pneumonia (B) Destruction and incomplete replacement of lung parenchyma and massive fibrosis (C) Masses composed of acid-fast organisms and necrotic tissue (D) Pathologic evidence of widely metastatic small cell lung cancer (E) Pleural-based tumors with encroachment into lung parenchyma

Internal Medicine

52.

A 58-year-old man comes to the clinic complaining of a rash on his legs that has developed over the past few weeks. The rash is itchy and painful and seems recently to have had some blistering. Additionally, he reports a history of severe arthralgias and near-debilitating fatigue. When asked about his past medical history, he tells you that he has been diagnosed with “some type of hepatitis.” Physical examination is remarkable for a palpable, purpuric rash over both lower extremities and hepatosplenomegaly. Routine laboratory studies show:

53.

A 40-year-old woman comes to the clinic complaining of bleeding from her gums and recurrent nosebleeds. These symptoms have been occurring off and on for the last few months. She has never had trouble like this in the past. Indeed, 1 year ago she had an elective cholecystectomy with no postoperative bleeding. She has been healthy all her life; she has had no prior hospitalizations and takes no medications. Physical exam, aside from mild gingival hypertrophy and scattered oral petechiae, is unremarkable. Laboratory studies show:

Blood, plasma, serum

Bleeding time

6.2 min

Sodium

136 mEq/L

Hemoglobin

13.0 g/dL

Potassium

3.8 mEq/L

Hematocrit

39.5%

Chloride

111 mEq/L

Leukocytes

7000/mm3

CO2

24 mEq/L

Platelets

12,000/mm3

Urea nitrogen

29 mg/dL

22 sec

Creatinine

2.1 mg/dL

Partial thromboplastin time

Glucose

109 mg/dL

Prothrombin time

10 sec

The patient’s creatinine level measured 1 year ago was 1.2 mg/dL. Given this patient’s presentation, which of the following is the most likely underlying condition? (A) (B) (C) (D) (E)

Hepatitis A Hepatitis B Hepatitis C Hepatitis D Hepatitis E

A review of the blood smear reveals reduced platelets with slightly increased volume but no other abnormalities or platelet clumps. With the exception of her platelet count, the patient’s lab work is essentially unchanged from preoperative labs taken 1 year ago. Which of the following is the most appropriate next step in management? (A) (B) (C) (D) (E)

Measure platelet autoantibodies Perform a bone marrow biopsy Provide a trial of oral corticosteroids Refer to surgery for splenectomy Start intravenous immunoglobulin

21

USMLE Step 2 Assessment Exam

54.

A 39-year-old man comes to the clinic for a routine annual physical examination. He has no current complaints and currently feels well. However, he mentions that a few weeks ago he passed out while on an airplane. He was on his way back from Mexico, where he had picked up a bad case of gastroenteritis. While waiting to use the lavatory, he suddenly blacked out and was apparently unconscious for 10 to 20 seconds, according to the flight attendant. He felt a little dizzy after the event and was mildly nauseous but in general reports being alert and oriented once he recovered consciousness. He refused to go the emergency room once the plane landed, and has had no recurrent episodes of syncope or gastroenteritis. He denies any history of such events, or of any seizures. A full review of systems, with emphasis on cardiac and neurologic symptoms, is unremarkable, as is the physical exam. An electrocardiogram is unremarkable. Which of the following is the most appropriate test to order for this patient’s syncope? (A) (B) (C) (D) (E)

55.

A concerned daughter brings her 89-year-old father to clinic for a checkup. His wife died about a year ago, and since then he has lived alone. He has been understandably grieved and reports having a diminished appetite. He cooks for himself and tries to stay involved in the local senior center. Over the past few months, however, he has suffered some disturbing skin changes. On his legs, numerous ecchymotic patches are evident. On close examination of his skin, hemorrhagic areas surround the hair follicles and the hairs themselves appear fragmented. Splinter hemorrhages are present in his nail beds, and the gums surrounding his few remaining teeth appear inflamed and mildly hemorrhagic. Coagulation studies are ordered but are unremarkable; the prothrombin time and partial thromboplastin times are within normal limits. A complete blood count, however, is remarkable for a hematocrit of 29% with normocytic indices. Which of the following is the most appropriate supplement to treat this patient’s symptoms? (A) (B) (C) (D) (E)

22

Exercise stress test Head CT scan No additional testing Signal-averaged electrocardiogram Upright tilt testing

Niacin Vitamin B6 Vitamin B12 Vitamin C Vitamin K

56.

A 45-year-old man is brought to the clinic with increasing confusion and difficulty ambulating. His deterioration has been pronounced over the past 2 years. He has no known medical issues and is not taking any medications. He has no allergies. On examination, he is noted to have pupillary accommodation but no reaction. His treponemal serology is positive, and his RPR titer is positive in the blood and the cerebrospinal fluid. Other laboratory studies are unremarkable. An MRI of his brain is unremarkable. Which of the following is the most appropriate treatment? (A) Ceftriaxone, 2 g daily for 2 weeks (B) Doxycycline, 100 mg twice daily for 7 days (C) Penicillin benzathine, 2.4 million units every week for 3 weeks (D) Penicillin G, 3 million units every 4 hours for 1 week (E) Penicillin G, 3 million units every 4 hours for 2 weeks

57.

A 45-year-old woman is admitted to the hospital for further evaluation of progressive shortness of breath with minimal exertion. The symptoms have been ongoing for the last year. The patient denies cough, fever, chills, palpitations, chest pain, or exposure to chemicals. She is moderately obese and has been trying to lose weight. She has no other medical issues and is not taking any medications. She has no drug allergies. Her husband reports that she has been snoring intermittently at night. Her vital signs are within acceptable limits. Lungs are clear. Heart rate and rhythm are normal. An echocardiogram reveals an elevated right ventricular systolic pressure, raising the issue of pulmonary hypertension. An electrocardiogram is unremarkable. All laboratory studies are within normal limits. Which of the following is the most appropriate next step in the management? (A) (B) (C) (D) (E)

Exercise stress test Left heart cardiac catheterization Lung biopsy Polysomnography test Right heart cardiac catheterization

Internal Medicine

58.

A 76-year-old man comes to the emergency department complaining of lower back pain. He is an ex-smoker and reports that he has a history of prostate cancer treated in another state 5 years ago. He is currently on no medications. Although the pain started out mild, he reports that it has been getting progressively worse over the last few days. Arising from a seated position makes the pain worse, and it has repeatedly woken him from sleep. He cannot seem to find a position that alleviates the pain. He also states that twice in the last 24 hours he has lost control of his bladder. Physical examination reveals a well-nourished elderly man in no apparent distress, but he has point tenderness in the upper lumbar region of his spine. Neurologic examination reveals some loss of motor strength in all muscle groups in both lower extremities. Plain spine films taken on arrival are read as unremarkable. Which of the following is the most appropriate next diagnostic study? (A) (B) (C) (D) (E)

59.

Bone scan CT scan of the brain Lumbar puncture MRI Serum PSA

A 25-year-old woman comes to the emergency department because of acute shortness of breath that started 4 hours before admission. She denies recent chest pain or dyspnea on exertion. She has no prior medical history. Her medications include ethinyl estradiol/levonorgestrel daily for birth control. She has a 10-year history of smoking a pack of cigarettes daily. Physical examination is notable for an anxious woman. Her blood pressure is 135/80 mm Hg, pulse is 110/min and respirations are 34/min. Lungs are clear. A ventilation-perfusion scan is at intermediate probability for a pulmonary embolus. Which of the following is the most appropriate management at this time? (A) (B) (C) (D) (E)

Administer aspirin Administer heparin Administer streptokinase Administer warfarin Place an inferior vena cava filter

60.

A 58-year-old woman is rushed to the emergency department after an overdose of pills. In an apparent suicide attempt, she swallowed handfuls of pills 2 hours ago. Judging from bottles found at her bedside, the pills included digoxin, aspirin, acetaminophen, oxycodone, and amitriptyline. Further, it seems she washed down the pills with a hefty dose of alcohol, as two empty bottles of vodka also were found. The patient is currently stuporous and difficult to arouse. A nasogastric tube is placed and the stomach is lavaged. Activated charcoal, followed by further lavage, then is started. In using activated charcoal in this patient, it is important to realize that charcoal does not affect the absorption of which of the following? (A) (B) (C) (D) (E)

61.

Acetaminophen Alcohol Amitriptyline Aspirin Digoxin

A 32-year-old man comes to clinic for a routine health check. He wants to have his vision checked because he thinks he is going blind. As an example, he says that twice in the last 6 months he has had car accidents driving home late from work. In fact, as he thinks about it, his vision seems worse at night, though he is convinced his vision is diminished in the daytime as well. His past medical history is significant for poorly controlled Crohn disease and an appendectomy 4 years ago. He takes sulfasalazine and is currently on a prednisone taper. Vital signs are within normal limits, and visual examination reveals a visual acuity of 20/30 OD and 20/20 OS. Examination of his conjunctivae reveals areas of keratinization and debris. His skin also shows areas of hyperkeratosis and seborrhea. Which of the following is the most likely cause of this patient’s symptoms? (A) (B) (C) (D) (E)

Vitamin A deficiency Vitamin B deficiency Vitamin D deficiency Vitamin E deficiency Vitamin K deficiency

23

USMLE Step 2 Assessment Exam

62.

A 54-year-old woman develops left flank pain of sudden onset with radiation to her inner thigh and labia. The pain is colicky and is accompanied by irritative voiding symptoms. Physical examination shows a patient in moderate distress, thrashing about on the stretcher, but who is otherwise noncontributory. Urinalysis shows microhematuria, and an emergency CT scan reveals the presence of a 4-mm calcified stone at the lower end of the left ureter, just above the junction with the bladder. She is given pain medication and is placed on intravenous fluids on the expectation that she will pass the stone. Three hours later she develops chills, a fever spike to 40.3 C (104.5 F), left costovertebral angle tenderness, nausea, and vomiting. A repeat urinalysis shows infected urine. Further management should include which of the following? (A) Addition of antibiotics to the current expectant therapy (B) Alkalinization of the urine and increased fluid intake (C) Emergency endoscopic removal of the stone (D) Emergency nephrostomy and intravenous antibiotics (E) Shock wave lithotripsy

63.

A 73-year-old man comes to the clinic for medical clearance before an elective surgical repair of a 6-cm abdominal aortic aneurysm. His cardiovascular history indicates that 8 months ago he suffered a transmural myocardial infarction (MI), from which he has recovered completely. He also gives a history of progressive, unstable, disabling angina that is no longer responding to medical management. Physical examination is noncontributory, and he specifically has no signs of congestive heart failure. His current electrocardiogram (taken at rest) shows evidence of his former infarction but is otherwise nondiagnostic. Before the proposed surgery is undertaken, it would be wise to do which of the following? (A) An echocardiogram (B) An exercise tolerance test and thallium scan (C) Perform cardiac catheterization and coronary angiogram (D) Treat him with diuretics and digitalis (E) Wait until 1 year has elapsed since his MI

24

64.

A 28-year-old woman has had difficulty climbing stairs over the past few days. She also notes tingling in her arms and legs. She has no chronic medical conditions; however, she had a “cold” 2 weeks earlier. Physical examination shows bilateral lower extremity weakness and mild weakness in the arms, intact sensation, and no deep tendon reflexes. Lumbar puncture shows elevated protein without an associated increase in cell count. Which of the following is the most appropriate initial treatment? (A) (B) (C) (D)

Glucocorticoids Interferon-beta Plasmapheresis Pyridostigmine

(E) Thymectomy

65.

A 59-year-old woman comes to the clinic with chest pain over the past few hours that is different from her typical angina because it is not relieved by rest. She has a history of diabetes mellitus and untreated hypercholesterolemia. When the clinic physician reaches her room and examines her, she is pain free and is telling the nurse about a new exercise routine with a personal trainer that she began earlier in the week. Her blood pressure is 160/90 mm Hg, pulse is 85/min, and respirations are 22/min. Examination reveals that she is diaphoretic and has an S3 gallop and bibasilar coarse rales. A chest radiograph shows mild pulmonary edema. An electrocardiogram taken on arrival to the clinic shows ST segment depressions in leads V3, V4, V5, and V6. Which of the following is the most likely diagnosis? (A) (B) (C) (D) (E)

Costochondritis Musculoskeletal pain Myocardial infarction Myocardial ischemia Pulmonary embolism

Internal Medicine

66.

A 58-year-old man has fatigue and occasional dizziness for the past 3 months. He has been told by family and friends that he looks pale. Physical examination confirms pale conjunctivae without icterus. His blood pressure is 145/86 mm Hg, and his pulse is 92/min. His hemoglobin level is 8 g/dL. Further laboratory studies indicate that he has microcytic anemia, with very low levels of serum ferritin. Which of the following is the most appropriate next diagnostic step? (A) (B) (C) (D) (E)

67.

Coombs test Hemoglobin electrophoresis Occult blood in stool Schilling test Serum creatinine

A 42-year-old man has been fired from his job because of inappropriate behavior. His employer insists that he has shown up for work drunk, an accusation that the patient vehemently denies. His case has not been helped by the fact that he has been arrested twice for driving under the influence of alcohol. In both cases the charges were dismissed because breath tests did not confirm illegal levels of alcohol. The patient admits that sometimes he feels as though he is drunk, he staggers when he walks, and his mind “is not clear.” On those occasions he also feels intense hunger, and eating food snacks seems to eliminate the symptoms. In response to direct questioning, he relates that these episodes never happen after he eats, but seem to be triggered by fasting. Except for mild obesity, his physical examination is completely normal. Which of the following would be most helpful to elucidate the diagnosis? (A) (B) (C) (D) (E)

68.

A 43-year-old woman has an episode of hematemesis and is brought in for evaluation. She is hemodynamically stable and her hemoglobin is 13 g/dL. Three years earlier she had been diagnosed elsewhere with peptic ulcer disease, and she has been to many physicians trying to regain her health. She has undergone two intensive trials of medical management that have been unsuccessful. On both occasions she took a full course of antibiotics to eradicate Helicobacter pylori and was treated with proton pump inhibitors. Discouraged by such failures, she became resigned to her fate and relied on over-the-counter antacids to alleviate her persistent epigastric pain. More recently she sought help for watery diarrhea, which initial studies characterized as secretory in nature; but the workup was never completed because she lost her medical insurance. At this time, an upper gastrointestinal endoscopy shows a 2-cm ulcer on the posterior wall of the first portion of the duodenum, with a fresh blood clot on it, but not actively bleeding. Beyond that ulcer, there is a second ulcer in the first portion and two more ulcers in the second portion of the duodenal loop. Which of the following is the most appropriate next diagnostic study? (A) (B) (C) (D) (E)

Biopsy of the small bowel mucosa Biopsy of the ulcer that bled Fasting serum gastrin levels Serum calcium levels Upper gastrointestinal series with barium

Fasting blood sugar, insulin, and C peptide levels Funduscopic examination Liver function tests MRI of the brain Psychiatric consultation

25

USMLE Step 2 Assessment Exam

69.

A 62-year-old man has massive hematochezia. He had a large bloody bowel movement 3 hours ago and two more bloody evacuations since then. On arrival at the emergency department he has a blood pressure of 78/60 mm Hg and a pulse of 110/min. He is given 2 L of Ringer lactate in 20 minutes, which brings his blood pressure to 100/85 mm Hg and his pulse to 105/min. This is followed by type-specific packed red cells as soon as they become available. In the first hour, he requires a total of four red cell packs to maintain stable vital signs. During that time, a nasogastric tube is inserted, which produces a clear, green fluid without blood. Anoscopy and proctosigmoidoscopy are also done, and except for a brisk flow of bright red blood coming from above the instruments, no pathology is detected and no bleeding lesion is identified. Which of the following is the most likely cause of his bleeding? (A) (B) (C) (D) (E)

70.

26

Colon cancer Diverticulosis Duodenal ulcer Inflammatory bowel disease Portal hypertension

An 81-year-old woman is brought to the emergency department with left lower quadrant pain and fever for 48 hours. She was discharged from the hospital 3 days ago after an admission for Escherichia coli urosepsis, for which she was successfully treated with gentamicin. Her temperature is 38.4 C (101.1 F), blood pressure is 162/102 mm Hg, and pulse is 88/min. Examination reveals abdominal tenderness and fullness in the left lower quadrant with guarding. An abdominal and pelvic CT scan shows an inflammatory mass surrounding the sigmoid colon in an area of many diverticula. Her leukocyte count is 18,400/mm3, hemoglobin is 13 g/dL, hematocrit is 40%, and a platelet count is 304,000/mm3. Her sodium is 132 mEq/L, potassium is 4.7 mEq/L, bicarbonate is 20 mEq/L, chloride is 110 mEq/L, BUN is 33 mg/dL, creatinine is 3.7 mg/dL, and glucose is 114 mg/dL. Which of the following is the most likely explanation for her renal insufficiency? (A) (B) (C) (D) (E)

71.

Acute tubular necrosis Diverticulitis Left ureteral obstruction Pyelonephritis Septic shock

A previously healthy 24-year-old woman has recurrent headaches. The symptoms began approximately 4 weeks ago, have been present almost daily, and are unrelated to time of day or meals. Her temperature is 37 C (98.6 F), blood pressure is 156/96 mm Hg, pulse is 78/min, and respirations are 14/min. Funduscopic examination is normal. There are no carotid bruits. Her lungs are clear. She has a regular heart rhythm without murmurs or extra heart sounds. On abdominal examination, a left upper quadrant bruit could be heard. She has no hepatosplenomegaly. Her pulses are full and symmetric in all regions. On two separate evaluations over the next week, her blood pressure measures between 155 and 170 mm Hg systolic and 95 and 105 mm Hg diastolic. There is no blood pressure gradient between her arms and legs. Which of the following is the most likely explanation for her hypertension? (A) (B) (C) (D) (E)

72.

Aortic dissection Coarctation of the aorta Hypertrophic cardiomyopathy Renovascular hypertension Takayasu arteritis

A 20-year-old college student develops a severe headache, nausea, photophobia, and myalgias. His temperature is 39.5 C (103.1 F), pulse is 105/min, blood pressure is 110/65 mm Hg, and respirations are 26/min. He appears acutely ill and is drowsy. Funduscopic examination is normal. There are no focal motor or sensory deficits. A lumbar puncture yields cloudy cerebrospinal fluid. A Gram stain of the fluid would most likely reveal leukocytes and which of the following organisms? (A) (B) (C) (D) (E)

Encapsulated yeast with hyphae Gram-negative bacilli in pairs Gram-negative cocci in clusters Gram-positive bacilli in pairs Gram-positive cocci in pairs

Internal Medicine

73.

A 62-year-old woman with a history of stable angina, hyperlipidemia, and a 35-pack-year tobacco history comes to the emergency department with substernal chest pain. The pain began 45 minutes ago at rest and has not been relieved by three sublingual nitroglycerin tablets. The pain is nonradiating and is associated with shortness of breath, nausea, and vomiting. Her blood pressure is 160/100 mm Hg and pulse is 100/min. Cardiovascular examination shows a left-sided S3. An electrocardiogram shows sinus tachycardia with 5-mm ST segment elevations in leads V1 to V6. While aggressive lowering of blood pressure and pulse is begun, which of the following is the most appropriate course of therapy? (A) Admission to the coronary care unit for exclusion of myocardial infarction (B) Immediate referral for emergent coronary artery bypass surgery (C) Immediate reperfusion with thrombolysis (D) Immediate synchronized cardioversion (E) PET scan of the myocardium to determine extent of ischemia

75.

A 32-year-old woman comes to the clinic one cold day in January complaining of fever, chills, headache, myalgias, and malaise. She also feels she is developing a cough and sore throat. Her symptoms seemed to have begun last night but were more severe this morning. She denies any major medical history and has never had any respiratory problems such as asthma or pneumonia. She has never been vaccinated for influenza. It is notable that many of her coworkers have been ill and have had similar symptoms. Physical exam reveals some mild oropharyngeal hyperemia, mild cervical adenopathy, and clear lung fields, though the patient has a cough on end expiration. The patient asks if there is any available treatment because she is extremely busy and “every day counts.” In addition to supportive care, which of the following is a treatment that will shorten the duration of this patient’s illness? (A) (B) (C) (D)

Amantadine if influenza titers are elevated Empiric macrolide antibiotics Empiric inhaled oseltamivir High-dose vitamin C and zinc lozenges

(E) No additional treatment is available 74.

A healthy 30-year-old man with an unremarkable history and physical examination has a PPD placed before starting medical school. Two days later, the skin at that site shows 8 mm of induration. He has never had a PPD placed before and has never been diagnosed or treated for tuberculosis. In addition, he has no risk factors for tuberculosis, was born and has lived his entire life in the United States, and has never received the bacillus Calmette-Guerin vaccine. Which of the following is the most appropriate next step in management? (A) No further management is needed (B) Prescribe isoniazid, rifampin, and pyrazinamide isoniazid for 6 to 12 months (C) Prescribe isoniazid for 6 to 12 months (D) Prescribe rifampin for 6 to 12 months (E) Refer him for bronchoscopy to confirm the result

27

Internal Medicine Assessment Exam Answers and Explanations ANSWER KEY 1. 2. 3. 4. 5. 6. 7. 8. 9. 10. 11. 12. 13. 14. 15. 16. 17. 18. 19. 20. 21. 22. 23. 24. 25.

C D C E E C E B E D E A B E B B E C A D C B B B C

26. 27. 28. 29. 30. 31. 32. 33. 34. 35. 36. 37. 38. 39. 40. 41. 42. 43. 44. 45. 46. 47. 48. 49. 50.

D A A A A C C C B C C C D E E B A E B C C E A E D

51. 52. 53. 54. 55. 56. 57. 58. 59. 60. 61. 62. 63. 64. 65. 66. 67. 68. 69. 70. 71. 72. 73. 74. 75.

B C C C D E D D B B A D C C D C A C B A D E C A C

29

USMLE Step 2 Assessment Exam

1.

30

The correct answer is C. This patient has hypertension and hypokalemia, two of the cardinal symptoms of primary hyperaldosteronism (Conn syndrome). He also has slightly high serum bicarbonate, suggestive of a metabolic alkalemia, which is also consistent with this diagnosis. Routine serum electrolytes and BUN/ creatinine are part of the workup of hypertension; this is done primarily to rule out the presence of hyperaldosteronism, Cushing syndrome, and intrinsic renal disease. Once the disease is suspected on clinical grounds, the test of choice to screen for Conn syndrome is a plasma renin activity to aldosterone ratio. A level greater than 30 in the appropriate clinical context is highly suggestive of the disease and warrants definitive testing that involves a volume challenge with normal saline followed by a repeat assay for aldosterone. Patients with confirmed hyperaldosteronism should have a full-body contrast CT scan to search for the tumor and should have it resected, if possible. An abdominal CT scan with contrast (choice A) would be helpful in looking for an abdominal mass, especially an adrenal one. Before such a search is undertaken, however, the presence of a metabolically active tumor must be confirmed. Adrenal and other incidentalomas are common and often are not the cause of the condition being investigated. A low-dose dexamethasone suppression test (choice B) is a common initial test for Cushing syndrome. In this test, the patient takes one 1-mg dose of dexamethasone at midnight and has a serum cortisol level drawn at 8 A.M. the following day. An A.M. cortisol of less than 5 µg/dL essentially excludes Cushing syndrome and is a good initial screening test for the disorder. In this patient, the negative review of systems and absence of physical findings other than hypertension make Cushing syndrome less likely than Conn syndrome. Renal biopsy (choice D) is a test best reserved for patients with elevated creatinine and abnormal urinalysis as part of the workup for intrinsic renal disease. A 24-hour urine cortisol (choice E) is the screening test of choice for patients suspected of having Cushing syndrome. The sensitivity of this test is poor, though, as is the specificity. Because excess cortisol exerts its hypertensive and potassium-wasting effect primarily through its action on the mineralocorticoid receptor (just like excess aldosterone), the syndrome can present similarly to hyperaldosteronism. Other physical findings, however, such as the buffalo hump, moon facies, purple abdominal striae, proximal muscle wasting, abdominal obesity, and mood changes are often present and should be actively sought during the routine history and physical examination for hypertension.

2.

The correct answer is D. This patient has COPD, or chronic obstructive pulmonary disease. The disease occurs in up to 15% of long-term (>20 pack-years) smokers and is a clinical diagnosis, though there are distinct histopathologic correlates. The diagnosis requires a chronic, productive cough of at least 3 months’ duration over 2 consecutive years. These patients have airtrapping that worsens as the disease progresses. The result is hyperinflated lungs. This manifests as an increase in TLC (total lung capacity) and FRC (functional residual capacity), not a decrease in FRC (choice A) and RV (residual volume), though there is often a decrease in VC (vital capacity). The ratio of FEV1 to FVC provides one means to distinguish between restrictive disease, such as interstitial lung disease, and obstructive conditions, such as COPD. In obstructive disease, the FEV1 decreases to a significantly greater degree than the FVC, resulting in a reduced FEV1/FVC ratio. In restrictive disease, there is an equivalent decrease in FVC (choice B), resulting in a normal ratio. This patient, with evidence of airway obstruction, such as prolonged expiratory phase of respiration, pursed lip breathing, and expiratory wheeze, would likely have a much lower FEV1 and FVC. The DLCO often decreases rather than increases (choice C) in COPD. Emphysematous changes result in alveolar surface loss, which reduces the area available for carbon monoxide diffusion. Conditions that occasionally increase DLCO include acute asthma and early congestive heart failure. Flow-volume loops, which are tracings representing a patient’s inspiratory and expiratory effort, can show evidence of restrictive or obstructive ventilatory defects. Restrictive, not obstructive, defects create a short and narrow tracing (choice E). The reduction of air and lung volumes results in a quick and short expiration. In contrast, obstructive defects are often wide and caved, representing a prolonged expiratory phase.

3.

The correct answer is C. This patient has evidence of an acute viral bronchitis. This condition, which accounts for 10 million office visits a year, is clinically defined as an acute respiratory illness lasting less than 3 weeks and characterized primarily by cough. The vast majority of cases are due to viruses and, in healthy hosts, do not require further workup. The acute onset of symptoms (less than a week) and sick-contact with a child who likely has a viral infection both argue in favor of a viral etiology. Symptomatic treatment includes antitussive agents such as dextromethorphan, decongestants, antipyretics, analgesics, and, if bronchial hyper-responsiveness is present (if the patient has a wheeze), inhaled beta-agonists.

Internal Medicine Answers and Explanations

It is not necessary to routinely order chest radiographs (choice A) for patients with a cough. If pneumonia is suspected, or if the patient has other risk factors (known lung disease, long tobacco history, immunosuppression, etc.), a radiograph is appropriate. In general, a chest radiograph should be ordered if any of the following criteria are met: pulse >100/min, respiratory rate >24/min, oral temperature greater than 38 C (100.4 F), an abnormal chest exam, or if there is an epidemiologic suspicion of SARS. Empiric macrolide antibiotics (choice B) are frequently used for patients with COPD who have a worsening of bronchitis. These patients have structural lung disease and altered ciliary functioning that increase their likelihood of having bacterial pathogens exacerbating their respiratory problems. Sputum culture and Gram stain (choice D) are not appropriate tests, given the high pretest probability of a viral etiology. Viral serologies and culture (choice E) are not appropriate because the results are usually not available until after the illness has resolved and are unlikely to change management of this apparent case of uncomplicated acute bronchitis. 4.

The correct answer is E. This patient suffers from Ménière disease, an idiopathic, episodic cause of vertigo and hearing loss. Multiple medications have proven ineffective in treating this, and it is usually very difficult to distinguish the natural waxing and waning of disease severity from a treatment response. Complete loss of vestibular sensation of balance (choice A) is a very rare disorder that requires extensive damage to the vestibular apparatus (the utricle, saccule, and semicircular canals). This is seen most commonly in oto/vestibulotoxicity secondary to aminoglycoside antibiotics or to loop diuretics. Experiencing complete resolution of symptoms (choice B) occurs occasionally in Ménière disease, especially if it is associated with an acute viral infection (labyrinthitis). If they truly had a case of chronic Ménière disease, though, only a minority has complete resolution. Experiencing a continued increase in the frequency and severity of vertigo symptoms (choice C) occurs only rarely in patients with Ménière disease. Observational studies have found that this occurs in only approximately 10% of patients. Development of refractory tinnitus (choice D) is an uncommon complication of Ménière disease. As with other manifestations, it stems from permanent damage to the cochlear hair cells. Over the long term it can even

induce neuropathic changes in the vestibulocochlear nerve and more central auditory nuclei. This can make the tinnitus refractory even to ablation of the vestibulocochlear nerve (central tinnitus). 5.

The correct answer is E. This patient has manifestations of systemic lupus erythematosus (SLE). Although anti-Sm antibody is positive in only approximately one quarter to one third of cases of SLE, it is highly specific for the condition; no other diagnosis routinely results in the presence of this antibody. This is in contrast to antinuclear antibody (choice C), which is highly sensitive but not very specific, being present in a variety of conditions. The presence of this anti-Sm is more common in African American females with SLE and has poor prognostic significance. It is often associated with CNS symptoms and lupus nephritis. Anticentromere antibodies (choice A) are associated with CREST syndrome, which manifests as calcinosis, Raynaud phenomenon, esophageal dysmotility symptoms, and telangiectasias. The test is not highly sensitive for the condition but is fairly specific for CREST syndrome. Antihistone antibodies (choice B) are sensitive indicators of drug-induced lupus, as they are present in most cases of the disease. Antihistone antibodies are also present in SLE, however, though they are neither a sensitive nor specific marker of the disease. Anti-Scl 70 (choice D), or antitopoisomerase-1 antibodies are highly specific, though not very sensitive, for systemic sclerosis. They are not specific for SLE.

6.

The correct answer is C. This patient has primary Raynaud phenomenon, that is, Raynaud phenomenon in the absence of any other rheumatologic disease. The symptoms are typical and the disease most commonly affects women between the ages of 20 and 50 years. The syndrome is caused by vasospasm of local arterioles, leading to pallor followed by cyanosis and pain. Eventually the vasospasm ceases and a reactive hyperemia occurs. Dihydropyridine calcium channel blockers are the most effective means of decreasing the symptoms of Raynaud phenomenon and resulted in symptomatic improvement over 4 to 12 weeks in six randomized controlled trials. In a prospective cohort, only 13% of patients who initially presented with isolated Raynaud phenomenon went on to develop a rheumatologic disorder such as lupus or scleroderma. The syndrome often remits on its own but should be treated symptomatically while appropriate workup for a rheumatologic cause proceeds.

31

USMLE Step 2 Assessment Exam

Amitriptyline (choice A) is a tricyclic antidepressant that at low doses is effective in treating neuropathy and neuropathic pain in a variety of diseases. It also has the side effect of orthostatic hypotension because of its action on alpha-1 adrenergic receptors in vascular smooth muscle. Its vasoactive properties, however, are not sufficient to be of use in Raynaud phenomenon, and neuropathy is not the cause of this patient’s pain. Gabapentin (choice B) is used widely for a variety of neurologic complaints, almost none of which have been studied in rigorous, controlled, randomized trials. Its effectiveness in vascular phenomena such as Raynaud phenomenon is unknown. Prazosin (choice D) is also a potent antagonist of the alpha-1 adrenergic receptor in vascular smooth muscle and is effective in the treatment of hypertension for this reason. It has been studied in the treatment of primary Raynaud phenomenon and was found to be ineffective in one small crossover randomized trial. Attacks were less frequent but were no less severe when compared with placebo. Verapamil (choice E) is a nondihydropyridine calcium channel blocker that has its primary effect on cardiac muscle and the cardiac SA and AV nodes. It has very little effect on peripheral vascular smooth muscle and has not been studied in the treatment of Raynaud phenomenon. 7.

32

The correct answer is E. This patient likely has Huntington chorea. Although the diagnosis is often difficult to make, the combination of chorea, neuropsychiatric symptoms, age of onset, and a family history of disease occurring in middle adulthood solidify the diagnosis. Genetic testing, which will reveal the evidence of trinucleotide repeats that produce a neurotoxic huntingtin protein, is available to confirm such a diagnosis. Abnormal copper regulation (choice A) is the cause of Wilson disease, not Huntington chorea. Wilson disease may also present with early behavioral changes, but is more likely to occur at a young age, does not result in movement disorders, and is associated with liver dysfunction. Dopaminergic loss in the substantia nigra (choice B), along with the presence of Lewy bodies, is associated with Parkinson disease. Parkinson disease manifests predominantly with “brady” symptoms, such as slowed gait, slowed thought, and a slow tremor. Chorea is the opposite of these bradykinetic symptoms, and is due to a relative excess of dopamine in the basal ganglia. Although patients with Huntington may have parkinsonian symptoms, the underlying cause is from trinucleotide repeats that result in primarily frontostriatal nerve destruction.

Elevated total body iron stores (choice C) are the cause of hemachromatosis. This disease will often cause heart failure, liver disease, and erectile dysfunction, but it does not commonly manifest as overt neuropsychiatric symptoms. Extracellular deposition of beta-amyloid and neurofibrillary tangles (choice D) is consistent with Alzheimer dementia, an unlikely diagnosis in this patient. Alzheimer tends to occur in older patients and presents with early memory impairment rather than movement disorder. 8.

The correct answer is B. Patterns of liver function tests tend to correlate with the pathophysiology of different disease states. Alcoholic hepatitis classically results in a transaminitis that is manifested as an AST at least twice the ALT. The ALT is usually less than 300 U/L and almost always less than 500 U/L. The important caveat is that if the patient already has severe cirrhosis or endstage liver disease, these ratios may not hold true. In contrast, viral hepatitis often has an AST level less than the ALT level, with both being greater than 1000 U/L (choice A). If the aminotransferase levels are severely elevated, however (in the 2000-3000 U/L range or greater) (choice C), another diagnosis should be considered. Shock liver and acetaminophen overdose are two common explanations for extremely high aminotransferase levels. In some cases of hepatitis, one can find near normal alkaline phosphatase and gamma-glutamyltransferase levels (choice D). Severe hepatitis and particularly alcoholic hepatitis, however, tends to result in elevations of both values. Often included in liver function tests are the total protein level and albumin level. The total protein is composed of albumin together with circulating immunoglobulin and various other carrier proteins. In liver disease, the albumin and thus the total protein are often low. If the total protein remains elevated and is more than 2 to 3 times the albumin level (choice E), another source of “excess” protein needs to be found. Often, disease states that cause chronic antigenemia or increased immunoglobulin production result in an increased total protein to albumin ratio. HIV infection, hepatitis B virus, and multiple myeloma are common causes of this abnormality.

9.

The correct answer is E. This woman has isolated hypercalcemia and a symptomatic urinary tract stone. Symptoms of hypercalcemia include those attributable to renal stones, as this patient currently has. Hypercalcemia also causes psychiatric problems such as

Internal Medicine Answers and Explanations

mood instability, anxiety, and cognitive problems, together with diffuse bone pain, constipation, and vague periumbilical abdominal pain. The most likely cause of hypercalcemia in young, otherwise healthy people is primary hyperparathyroidism, usually secondary to a parathyroid adenoma. Chronic diarrhea (choice A) would not be an expected symptom. High levels of serum calcium cause decreased smooth muscle tone and autonomic dysfunction in the gut, causing constipation rather than diarrhea. Exertional dyspnea (choice B) has not been described as a symptom of hypercalcemia. Patients often have mild generalized weakness that improves after correction of the cause of hypercalcemia. Swollen, painful joints in the fingers and toes (choice C) are symptoms of osteoarthritis, which has not been connected with hypercalcemia. Primary hyperparathyroidism often causes diffuse bone pain because of rapid bone turnover and can rarely lead to cases of pseudogout because of precipitation of calcium pyrophosphate crystals in joints. Joint manifestations are rare, however, and pseudogout typically involves the large joints such as the knees and hips rather than the fingers and toes. Muscle tetany (choice D) is a finding in hypocalcemia, not hypercalcemia. This finding is known as Trousseau’s sign and results from the combination of hypocalcemia and ischemia caused by inflating the blood pressure cuff. Another finding in hypocalcemia is Chvostek’s sign, in which percussion over the facial nerve causes twitching of the corresponding muscles of the face. Physical examination of patients with known hypercalcemia is usually completely unrevealing. Even palpation of the thyroid bed for adenomas seldom reveals the causative parathyroid adenoma. 10.

The correct answer is D. Glucose creates a powerful osmotic force that can pull water out of cells and into the intravascular space, resulting in a dilutional hyponatremia. In cases of severe hypoglycemia, this dilutional effect can result in significant hyponatremia, at least by laboratory testing. The sodium level needs to be adjusted for the elevated blood glucose. Basically, for every 100 mg/dL above 100, the sodium level decreases by 1.6 mEq/L. The adjusted serum sodium, then, is: Measured sodium + (glucose – 100)/100 × 1.6 = corrected serum sodium Albumin (choice A) needs to be measured to determine the true serum calcium. Because calcium is tightly protein bound, the total level of calcium increases and decreases with albumin levels, whereas the ionized, active portion stays constant.

Alcohol (choice B) can cause a ketoacidosis but usually causes a hypoglycemia. It is metabolized rapidly by cells and generally is not an effective osmole. It is unlikely to explain this patient’s sodium level. The creatinine level (choice C) is important to know, as insulin is cleared renally. For nondialysis patients, however, this value does not assist in understanding the patient’s hyponatremia. Potassium (choice E) needs to be followed closely when treating hyperglycemia because patients can become hypokalemic quickly. This value does not affect the serum sodium level directly, however. 11.

The correct answer is E. He passed a uric acid stone and the remaining stone that he has not passed yet has the radiologic characteristics of another uric acid stone. Uric acid stones can be dissolved medically by alkalinization of the urine. Acidification of the urine (choice A) is the opposite of what will dissolve the stone. Endoscopic extraction (choice B) or shock wave lithotripsy (choice D) does not need to be considered yet, because more conservative medical management can be expected to solve the problem. Antibiotic therapy (choice C) has a role to play in the management of struvite stones in which chronic infection of the urinary tract is an etiologic factor.

12.

The correct answer is A. This patient likely has primary sclerosing cholangitis (PSC), an autoimmune-mediated inflammation and destruction of the large bile ducts that results in fibrosis and stricture. It is associated commonly with inflammatory bowel disease, and although partially treatable with ursodeoxycholate, is relentlessly progressive. Definitive treatment is liver transplantation. All that is biliary is not gallstones. Recurrent gallstones, either cholesterol (choice B) or pigment (choice E), are a less likely cause of this patient’s symptoms than is PSC. Although he has no major risk factors for gallstones, he does have a major risk factor for PSC, together with having a serum marker of autoimmune disease, a positive ANCA. Often an antinuclear antibody assay and an antismooth muscle antibody test also are positive. This patient is at increased risk for colon cancer (choice C), though Crohn disease is less associated with early colon cancer than it is with ulcerative colitis. This patient, given an unremarkable review of symptoms, is more likely to have PSC than a large, metastatic colon cancer causing biliary obstruction. There is no reason to suspect a hepatoma (choice D), which is unlikely to present primarily as a cholestatic picture.

33

USMLE Step 2 Assessment Exam

13.

14.

34

The correct answer is B. This patient has an increase in heartburn symptoms and abdominal pain. Dyspepsia is a common complaint, and the most common causes are gastroesophageal ulcers/erosions, atypical gastroesophageal reflux disease, nonulcer dyspepsia, and gastric/duodenal cancer. The diagnostic concern here is gastric cancer, the incidence of which increases with age. In patients older than age 45 years, endoscopy is recommended as the initial screening test of choice. It has the advantages of excellent sensitivity, the ability to perform biopsies for ulcers and Helicobacter pylori testing, and potential therapeutic intervention if an ulcer is found to be bleeding. If an ulcer or other benign pathology is defined, the patient can be given antibiotics for H. pylori eradication or can be given a protonpump inhibitor for his symptoms. Empiric therapy with a proton pump inhibitor (PPI) (choice A) is only acceptable for younger patients whose risk for gastric cancer is very low. In these patients, response to a PPI is adequate to make the diagnosis of dyspepsia, and this therapy is likely to cause healing of any ulcers that are present. Testing for H. pylori through serology or stool antigen may be of benefit, as eradication of the organism with antibiotics leads to a better chance of resolving the symptoms. Should a younger patient fail to respond completely to this therapy or if they have warning symptoms such as bleeding, fatigue, weight loss, or dysphagia, they should be referred for early endoscopy. Fecal occult blood testing (choice C) helps assess whether the patient is losing blood from a gastroduodenal ulcer or a gastric cancer. If the patient is not actively bleeding from such a lesion, however, this test is falsely negative. Again, in older patients with new-onset dyspepsia, the concern for gastric cancer merits an invasive workup with EGD. Serology for H. pylori (choice D) is an appropriate test in younger patients and helps to determine whether eradication therapy is needed. It provides no information, however, about other potential causes of this patient’s pain. An upper gastrointestinal series (choice E) is incorrect because of its low sensitivity for gastric cancer, especially for early lesions that may be amenable to therapy. Its sensitivity for ulcers is also fairly low and does not allow biopsies to be performed if a lesion is found. The correct answer is E. This patient has chronic myelogenous leukemia, definitively diagnosed by the presence of the Philadelphia chromosome, t(9;22). Medical treatment options are varied, and the exact protocols are complex and evolving. However, young patients

with available donors and no major comorbidities do better with stem cell transplant than with pure chemotherapy. As such, the definitive treatment for this patient is hematopoietic cell transplantation from an HLA-matched donor. Cytarabine and busulfan (choice A), as well as hydroxyurea (choice B), are chemotherapeutic agents that can be combined with interferon-alpha (choice C). Interferon-alpha–based regimens are highly effective treatments that can induce remission. Recently, oral tyrosine kinase inhibitors have been found to be superior to some of these traditional treatments. However, in young patients, stem cell transplantation is still the definitive treatment. Interferon-beta (choice D) is not used to treat CML. 15.

The correct answer is B. An isolated GGT elevation may be the only laboratory abnormality seen in a clinically stable alcoholic patient. Although the test is by no means sensitive enough to use for screening purposes, an isolated GGT level (relative to alkaline phosphatase and other liver function tests) should raise the possibility of alcohol abuse. This patient was recently fired from work (raising the question why), reports depressive symptoms, and has rhinophyma, a physical finding loosely associated with alcohol use. Other drugs that can cause an isolated GGT elevation include barbiturates and phenytoin. Acetaminophen (choice A) toxicity will present with massive transaminitis, with levels in the thousands. An isolated GGT elevation, which can indicate early liver disease, is not consistent with acetaminophen toxicity. In many patients, amiodarone (choice C) can cause a moderate increase in serum transaminases and, rarely, more severe hepatic injury. Hepatomegaly is usually present, and a transaminitis, not an elevation of cholestatic markers such as GGT, is more likely. Atorvastatin (choice D), like all statins, can cause elevations of liver enzymes, though they often return to normal. Again, the pattern is a transaminitis, not an isolated elevation of GGT. Metformin (choice E), which acts primarily by limiting hepatic gluconeogenesis, needs to be used cautiously in patients with liver failure. It is not, however, commonly associated with hepatic abnormalities.

16.

The correct answer is B. This patient is not stable. Altered mental status, hypotension, and a weak, thready pulse warrant immediate action. The first intervention in ventricular tachycardia is early defibrillation. The patient should be shocked with 3 shocks. After these stacked shocks, other medications such as amiodarone

Internal Medicine Answers and Explanations

(choice A), epinephrine (choice C), or lidocaine (choice D) can be used. The mnemonic for remembering the sequence of events in treating unstable v-tach is, “Shock, shock, shock, everybody shock, and let’s make patients better.” This stands for three stacked shocks, epinephrine, shock, then amiodarone, lidocaine, magnesium, and procainamide. This patient is not in a stable rhythm, and should be treated now. Given that he is already on a cardiac monitor, and assuming a crash cart is available, not much will be gained by an immediate transfer to a cardiac care unit (choice E). 17.

The correct answer is E. This patient likely has pernicious anemia, a disease characterized by autoimmune destruction of the parietal cells that secrete intrinsic factor, which is necessary for vitamin B12 absorption. The “seven clinical Ps” of pernicious anemia are pancytopenia, peripheral neuropathy, posterior spinal column neuropathy, pyramidal tract signs, papillary (tongue) atrophy, pH elevation (gastric fluid), and psychosis (megaloblastic madness). The classic diagnostic test to confirm the clinical suspicion of pernicious anemia is a Schilling test. Basically, radiolabeled cobalamin is administered orally after a parenteral dose has saturated all the cobalamin binding sites. The urine is then checked for excreted radiolabeled cobalamin. If there is cobalamin in the urine, then absorption is adequate and the patient is unlikely to have pernicious anemia. If there is no absorption (i.e., no urinary cobalamin), another dose of cobalamin is given, this time with coadministration of intrinsic factor. If there is absorption this time, then the diagnosis of pernicious anemia (which is due to a deficiency of intrinsic factor) is almost certain. If there is no absorption, small gut pathology (Crohn, ileal resection, bacterial overgrowth, etc.) should be suspected. A bone marrow biopsy (choice A) will show erythroid hyperplasia, megaloblastic cell lines, and normal or high iron, but is not necessary for the diagnosis of this disease. Although it can point toward folate and B12 deficiencies (but cannot differentiate the two), it will not diagnose the cause of the deficiency. The Ham test (choice B) is an acidification test used to test for paroxysmal nocturnal hemoglobinuria, a rare cause of hemolytic anemia. It is not an appropriate test for this patient, who does not appear to have a hemolytic anemia. Lumbar puncture (choice C) and MRI (choice D) can be useful if this patient’s primary problem is believed to be a CNS lesion, such as demyelination from multiple sclerosis. However, the peripheral blood smear should

point toward a hematologic cause of this patient’s problems. As such, these are not the most appropriate first tests to order. 18.

The correct answer is C. This patent has a chylous effusion, defined as a lymphatic leak into the pleural space. There are a variety of causes of chylous effusions, falling mainly into the categories of traumatic and nontraumatic. Traumatic causes include surgery, in which the thoracic duct can be nicked (this patient has no surgical history) and thoracic trauma. Nonsurgical causes include infection and, much more commonly, malignancy. The most common malignancy to cause a chylothorax is lymphoma, as hinted at by this patient’s B-symptoms and abnormal chest radiograph. There is no reason to suspect an acid-fast organism (choice A) such as tuberculosis (TB). Such infections rarely cause a chylous effusion and, although some of this patient’s symptoms may be attributed to TB, he has no cough or evidence of TB on chest radiograph. Lymphangioleiomyomatosis (choice B) is an extremely rare disease that affects women of childbearing age and may cause chylous effusions. It is an unlikely diagnosis in this middle-aged man. Pancreatic disease, including adenocarcinoma (choice D), can cause pleural effusions and ascites, though it does not cause chylous effusions. This patient has no risk factors for mesothelioma (choice E), which is associated almost exclusively with asbestos exposure. Further, such a condition, although it may cause an exudative malignant effusion, is unlikely to cause a chylous effusion.

19.

The correct answer is A. Although a radioactive uptake scan is not necessary to diagnosis this patient’s condition, subacute thyroiditis, it is a common way to think of the pathogenesis of hyperthyroidism. Conditions that increase thyroid activity, such as Graves disease, can be expected to result in increased incorporation of iodine and will result in increased uptake of the radioactive tracer. Conditions that result in increased thyroid hormone without increasing the production of hormone, basically exogenous hormone (sometimes taken for weight loss), or inflammatory damage of the thyroid and release of colloid, result in reduced uptake. The excess thyroid hormone results in feedback inhibition of TSH, resulting in less uptake of iodine. Subacute thyroiditis, in which thyroxine is “spilled” from an inflamed thyroid gland, can result in frank hyperthyroidism with a reduced uptake of radioactive iodine. A focal area of increased uptake with the rest of the gland revealing reduced uptake (choice B) is consistent

35

USMLE Step 2 Assessment Exam

pumps, stabilizing the cell membrane. It is not appropriate in this patient with mild hypokalemia and no classic EKG changes. Cardiac catheterization (choice B) and heparin (choice D) are not appropriate. This is not an acute coronary syndrome and should not be treated as such. Further, if there is a pericardial effusion, heparin can place the patient at an increased risk for tamponade from hemorrhage. Note that troponin levels are often elevated in renal insufficiency. Indomethacin (choice E) is appropriate for many causes of pericarditis but not for uremic pericarditis. Dialysis is the definitive treatment. Further, whatever residual renal function this patient has can be destroyed by NSAIDs, which reduce the glomerular filtration rate by inhibiting afferent arteriole dilatation, a prostaglandin-mediated event.

with a toxic adenoma. The adenomatous tissue secretes thyroxine free from TSH-mediated feedback. The thyroxine reduces TSH secretion, however, which does affect the normal portion of the gland. Without TSH, the rest of the gland does not incorporate iodine readily. Multinodular toxic goiter presents similarly, but with multiple nodular areas of increased uptake surrounded by areas of reduced uptake (choice D). Increased uptake throughout the gland (choice C) indicates an increased amount of colloid formation, as would be seen in conditions such as Graves disease, not subacute thyroiditis. Given this patient’s symptoms and abnormal thyroid function tests, a normal scan (choice E) is extremely unlikely. Given a low TSH level, at least part of the gland will act abnormally, as the stimulus to incorporate iodine is not present. 20.

The correct answer is D. In all states, hepatitis A is a reportable illness. Given the ease of transmission, and that most patients can transmit the disease to family members, public health officials need to be notified sooner rather than later. This patient should have immunity to hepatitis A, and does not appear to have risk factors for recurrent infection in any case. In most patients, IgM levels are detectable 5 to 10 days before symptoms, with IgG levels rising later in the course of the infection. Hepatitis A vaccination (choice A) is not necessary. Similarly, inasmuch as this patient already has hepatitis B antibodies, hep B vaccination (choice B) is not necessary. Liver biopsy (choice C) is inappropriate because hepatitis A, unlike B and C, does not cause chronic disease. Liver biopsy is used in some risk-stratification algorithms for approaching the diagnosis and treatment of hepatitis C. In addition, some physicians use a right upper quadrant ultrasound and measure alpha-fetoprotein levels (choice E). Again, given the natural course of hepatitis A, this degree of testing is inappropriate.

21.

The correct answer is C. This patient has uremic pericarditis. The “squeaky” noise heard on exam is a classic three-phase friction rub. Uremic pericarditis, unlike other forms of pericarditis, is not associated with classic pericarditis EKG findings, such as diffuse ST elevations and PR depressions. Although the pathophysiology of uremic pericarditis is unclear, it is related to toxin buildup from reduced renal function and is thus an indication to start dialysis in a patient who might otherwise have been watched closely. Calcium gluconate (choice A) is used for severe hyperkalemia because it serves to bind to Na/K membrane

36

22.

The correct answer is B. This patient’s pain seems to be esophageal in origin. GERD and esophageal spasm are common causes of noncardiac chest pain and often have atypical presentations that mimic coronary chest pain closely. Esophageal spasm often responds to nitroglycerin, which can relax the esophageal smooth muscle. Intermittent dysphagia associated with chest pain and a history of GERD strongly suggests esophageal dysfunction. In this setting, a pH probe and outpatient manometry is an appropriate next diagnostic step. Cardiac catheterization and angiography (choice A) are not appropriate. This patient’s only cardiac risk factor is hypertension. Given her young age and completely normal cardiac evaluation, cardiac catheterization is unlikely to be a high yield study and is also highly invasive. Similarly, given the normal thallium perfusion scan, further imaging of the heart with a transthoracic echocardiogram (choice C) is unnecessary. If this patient were not on GERD therapy, an 8-week trial of a proton pump inhibitor (PPI) would be an appropriate first step in the management of suspected esophageal chest pain. Given that this patient is already on a PPI, the addition of an H2 blocker (choice D) is unlikely to be beneficial. Were the dysphagia symptoms constant, endoscopy (choice E) would be an appropriate first step. Routine endoscopy in the evaluation of suspected esophageal chest pain, however, is not recommended. Only 10% of patients have esophagitis, and conditions such as GERD or spasm are better evaluated with other modalities.

23.

The correct answer is B. Desmopressin is an antidiuretic hormone (ADH) analogue that can be used to treat central diabetes insipidus (DI), the cause of this

Internal Medicine Answers and Explanations

patient’s hypernatremia. Hypernatremia in the setting of neurosurgical procedures affecting the pituitary or hypothalamic tract warrants an evaluation for DI. Mild cases of DI often resolve spontaneously, as the hypothalamic-pituitary access regains function. Severe damage to the pituitary, however, may result in a triphasic response. Initially the patient may have abrupt onset of DI, which then seems to resolve slowly. This is caused by an initial inhibition of ADH release, followed by inappropriate ADH response from a degenerating posterior pituitary. Finally, as stores of ADH are depleted, the patient once again presents with a picture of DI. As such, DI needs to be followed closely with serial sodium levels. If one is suspicious of the diagnosis, it can be confirmed with a water restriction test. Water restriction in a patient with normal homeostatic mechanisms should result in an increase in urine osmolality as the body attempts to retain free water. If the increase is inappropriately minimal and then improves with vasopressin (ADH), the diagnosis of central DI is highly likely. Demeclocycline (choice A) is a tetracycline-type antibiotic that blocks ADH, and is used, understandably, for treating the syndrome of inappropriate antidiuretic hormone (SIADH) release. It would likely worsen this patient’s condition. Furosemide (choice C) is a loop diuretic. Patients tend to lose more water than salt. Indeed, the combination of salt tablets and loop diuretics can be used to treat hyponatremia caused by SIADH. As such, it should not be used to treat hypernatremia. Thiazides, which lose relatively more salt than water, are used occasionally to treat SIADH and can be quite effective. Free water can be given to attempt to dilute the hypernatremia. Unfortunately most is wasted by the kidneys because of the lack of ADH in the distal tubules. The administration of salt, present in hypertonic (choice D) and normal saline (choice E), should be avoided, as this can worsen the hypernatremia. 24.

The correct answer is B. This patient has severe aplastic anemia. This may be from a transient infection, or the antibiotic she was given may have caused it. Chloramphenicol is very common in third world nations. Although it is an extremely effective antibiotic (and is used often for nasty gastrointestinal infections like typhus), it has a high incidence of aplastic anemia. A bone marrow biopsy, which can help determine the cause of the aplastic anemia, is the next diagnostic step in this patient’s workup. In the meantime, she needs a transfusion.

Blood and stool cultures (choice A) are not necessary. This patient does not seem to be infected at this time. She has no fever, localizing signs, or evidence in her history that the illness persisted after treatment. For similar reasons, a CT scan of the abdomen (choice C) is not necessary at this time. If there is an infection in the marrow itself, it will be seen on bone marrow biopsy. An echocardiogram (choice D) is not necessary. This patient’s murmur, consistent with a high velocity flow murmur, will probably resolve with transfusion. An erythropoietin level (choice E) can provide indirect evidence of bone marrow function. Cytologic examination (and culture), however, is necessary to determine if this patient has lymphoproliferative disorder, drug-induced toxicity, viral suppression, or marrow replacement. 25.

The correct answer is C. This patient has paroxysmal nocturnal hemoglobinuria (PNH), a cause of hypercoagulability and hemolytic anemia that is intermittent and difficult to diagnosis. A defect in the hemopoietic cells renders them vulnerable to complement-mediated lysis, which is precipitated by even a mild acidosis, such as the mild respiratory acidosis that may occur during sleep. The increased activation of complement also is accompanied by platelet aggregation, enhanced expression of tissue factor, and impaired fibrinolysis, all of which place these patients at risk for recurrent clots, particularly in the hepatic and intra-abdominal organs. The presence of PNH should be suspected in any patient with unexplained or intermittent hemolytic anemia, particularly if accompanied by recurrent venous thrombosis and pancytopenia. An appropriate diagnostic test is a Ham’s acidification test. The red blood cells are exposed to an acidic solution that activates complement. Abnormal cells lyse, whereas normal ones do not. Although the anemia and leukopenia seen in these patients are complement mediated, complement levels are not a useful test. Unless there is significant, ongoing hemolysis, they will not be markedly abnormal. Further, the measurement of complement levels (choice A) is a nonspecific test, unlike a Ham test. Gel electrophoresis (choice B) can be used to look for abnormal cell membrane proteins and sometimes is used to confirm conditions such as hereditary spherocytosis. This technique is not used commonly to diagnosis PNH. A lactate dehydrogenase level (choice D) can provide further evidence of hemolysis if elevated but does not assist in the diagnosis of this patient’s condition.

37

USMLE Step 2 Assessment Exam

An osmotic fragility test (choice E) is used to test for hereditary spherocytosis. The abnormal morphology is seen easily on a peripheral smear, not seen in this case. The abnormal cells, which have a reduced surface area relative to volume, lyse when placed in a hypotonic fluid. 26.

27.

38

The correct answer is D. This patient has hereditary spherocytosis, the most common hemolytic anemia caused by a red blood cell membrane defect. An elevated MCHC and RDW combined with the finding of spherocytes are almost pathognomonic for the condition. A positive osmotic fragility test further solidifies the diagnosis. In this test, the spherocytes, with an already reduced surface area to volume ratio, lyse when placed in a hypotonic solution. Patients often present in childhood, but mild or intermittent cases may be picked up in young adults. The definitive treatment is splenectomy. Hereditary spherocytosis is caused by a genetic defect resulting in abnormal cell membranes that the spleen then destroys. It is not a classic autoimmune disease, and thus should not be treated with corticosteroids (choice A). These patients have an appropriate reticulocytosis and are unlikely to benefit from additional exogenous erythropoietin (choice B). Further, the definitive treatment is to stop the hemolysis, not replace the lost cells. As such, splenectomy is the most definitive treatment. If this patient has a mild case or does not want surgery, then no additional treatment (choice C) is necessary. These patients should take supplemental vitamins, particularly folate, to help with the hemolytic anemia, and may need transfusions if the anemia becomes symptomatic. Stem cell transplant (choice E) is a dramatic intervention that this condition does not warrant. Although in theory it can treat hereditary spherocytosis, patients respond well to a safer and more cost-efficient splenectomy. The correct answer is A. This patient has a classic presentation of atrioventricular nodal reentrant tachycardia (AVNRT). Regardless of whether you recognize the classic EKG findings (initiation by an atrial premature beat; pseudo-R waves representing a buried, late P-wave), you should recognize the treatment for a stable, supraventricular tachycardia. The first intervention should be a vagal maneuver, such as direct pressure on the eyes, carotid massage, a Valsalva maneuver, or sudden immersion in cold water (to name a few). If these interventions fail, as they often do, a small bolus of adenosine, which can be doubled and repeated, is the next appropriate intervention. Adenosine will block the

AV node temporarily. If the rhythm is supraventricular, the atrial rhythm will be easier to see. If the rhythm is nodal in origin, it can be suppressed, allowing for the native rhythm to take over. Amiodarone (choice B) does have nodal blocking properties but is usually not necessary for AVNRT, which can be chronically suppressed with selective nodal blocking agents that have a safer side-effect profile. Amlodipine (choice C) is a calcium channel blocker that acts mainly on the peripheral vasculature. If adenosine fails, a nondihydropyridine calcium channel blocker such as diltiazem or verapamil can be used. DC cardioversion (choice D) is appropriate if the patient is unstable. This patient, although uncomfortable, is maintaining an adequate blood pressure. Digoxin (choice E) can block the atrioventricular node, mainly through increased vagal activity. Adenosine, with much easier dosing and a much shorter half-life, is a much more appropriate acute therapy. Digoxin is reserved as an option for chronic treatment of AVNRT, and SVT in general. 28.

The correct answer is A. This patient has new-onset congestive heart failure. Her history is very clear for the telltale signs of increasing left heart filling pressures: paroxysmal nocturnal dyspnea, orthopnea, decreased exercise tolerance, and pulmonary congestion. Given her age and track record of never having been hospitalized, the correct course of action is to obtain an echocardiogram as an inpatient and then begin a medication regimen while in the hospital. Limiting her activity (choice B) and salt intake (choice C) are not correct because we fail to acknowledge the presence of a clear process occurring in this patient. Simply prescribing a diuretic (choice D) assumes that you understand that she has CHF, but given our lack of diagnostic evidence for this, and with no way for us to evaluate the efficacy of the dose prescribed in alleviating her symptoms, it is unacceptable to send this patient away with such a regimen. Referring for an echocardiogram and re-evaluating in 3 months (choice E) is unacceptable, given that the interval of time between her echocardiogram and initiation of therapy could result in this patient incurring severe morbidity or even death.

29.

The correct answer is A. Niacin deficiency can cause pellagra, which presents with dermatitis, dementia, and diarrhea, among other symptoms. The dermatitis is classically in sun-exposed areas and may be mistaken for a sunburn. It is extremely rare in developed countries, where it tends to occur mainly among alcoholics.

Internal Medicine Answers and Explanations

In less developed countries, where diets may consist of mainly low-niacin-content foods (cereals, corn, and sorghum), it is still endemic. Common findings include symmetric hyperpigmentation, a red tongue, diarrhea and vomiting, and neurologic symptoms, including anxiety, disorientation, dementia, and encephalopathy. Thiamine deficiency (choice B) is associated with beriberi and Wernicke-Korsakoff syndrome. Beriberi may be “dry” or “wet.” Dry beriberi refers to neurologic deficits due to chronic thiamine deficiency, commonly a symmetric peripheral neuropathy. Wet beriberi includes these symptoms as well as cardiomyopathy. Although this patient may benefit from some thiamine, its deficiency does not explain his symptoms, which are most likely attributable to niacin deficiency. Findings associated with a deficiency of pyridoxine, commonly known as vitamin B6 (choice C), include peripheral neuropathies, seizures, dermatitis, and glossitis. Because pyridoxal phosphate is involved in heme synthesis, a microcytic anemia is also seen. Vitamin B12 (choice D), or cobalamin, deficiency can cause a macrocytic anemia and neurologic changes, and is classically associated with pernicious anemia. Dietary B12 deficiency is rare and is more likely to be present in the elderly or those on fad diets. Although this patient may well benefit from any vitamin supplement, niacin is the most appropriate supplement to treat his symptoms. Scurvy, caused by a deficiency of vitamin C (choice E), does not explain this man’s symptoms. Without adequate vitamin C, dysfunctional collagen results in capillary fragility, poor wound healing, and abnormal hair development. Patients will often be subject to gum disease and may lose teeth as the collagen mesh of their gums breaks down. Ecchymosis and hematomas are often present. 30.

The correct answer is A. Influenza vaccine is made from inactivated virus and is safe for pregnant patients. Given that pregnant patients are at risk for particularly morbid cases of influenza, all patients without other contraindications should be vaccinated during the second or third trimester. Patients with significant comorbidities should be vaccinated immediately, regardless of trimester. Oseltamivir (choice B) and amantadine (choice C) are probably safe in pregnancy (Class C FDA Pregnancy Rating) but are less effective, more expensive, and not as well tolerated as a vaccination. This patient would benefit more from an immunization at this clinic visit than from these medications. There is no compelling reason to discourage breastfeeding (choice D) or to delay the immunization until the postpartum period (choice E). The vaccine is made

from killed viruses and is not considered transmissible. Further, given the risks of influenza during pregnancy, where it can result in a high rate of hospitalizations, there is no reason to delay immunization. The ideal time to immunize is in October and November, to allow time for the body to build antibodies before flu season. 31.

The correct answer is C. This patient may have primary HIV infection, which can present weeks to months after exposure and presents as an acute viral illness. A high degree of suspicion is needed to make the diagnosis, inasmuch as the signs and symptoms are not specific. Diffuse (as opposed to regional) lymphadenopathy is present in 40 to 70% of patients. Mucocutaneous ulcers are further suggestive of the diagnosis, as is the time course of the illness. This primary infection correlates with high degrees of viremia (so the appropriate test is to determine the HIV RNA level), although the body may not have had time to produce adequate amounts of antibodies for traditional screening tests such as ELISA (choice B) and the confirmatory Western blot (choice E). A CD4+ T-cell count (choice A) is used to monitor disease progression but not to diagnose the disease. P24 antigen serologies (choice D) have a sensitivity of 75 to 95% and were the early-infection test of choice before the widespread availability of polymerase chain reactionbased RNA tests. Given that the viral load is exceedingly high during primary HIV infection (often greater than 100,000 copies per milliliter), HIV RNA levels are the most sensitive diagnostic test for primary HIV.

32.

The correct answer is C. When evaluating an elevated alkaline phosphatase level, it is important to determine if it is due to cholestatic liver disease or to bone disease. Alkaline phosphatase is present in a variety of tissues, including bone, liver, intestine, and prostate, but is usually significantly elevated only in bone and liver disease. Some laboratories have the ability to check liver specific isoenzymes of alkaline phosphatase; the alternative is to check another marker of cholestatic liver disease that closely correlates with it. Gamma-glutamyl transpeptidase (GGT) is elevated in cholestatic liver disease and may be even more sensitive than alkaline phosphatase. If the GGT is normal, it is reasonable to infer that the elevated alkaline phosphatase is from bone. This patient, who is basically asymptomatic, may have Paget disease, a condition of increased bone turnover that may present with increased risk of fractures, hearing loss and other cranial nerve involvement, and an increased risk of osteosarcoma, or simply abnormal radiographic findings of sclerosis. Given this patient’s likely diagnosis, there is no need to repeat the alkaline phosphatase test (choice E); there is no reason

39

USMLE Step 2 Assessment Exam

to suspect laboratory error or a fluctuating enzyme level. Tests of hepatocellular injury, such as alanine aminotransferase (choice A) and aspartate aminotransferase (choice B), are not always elevated in cholestatic disease and are thus ideal markers to distinguish the source of an elevated alkaline phosphatase. Lactate dehydrogenase (choice D) is present in tissue throughout the body, including the liver, and is probably most useful as a marker for general cell death and hemolysis and for myocardial infarction in labs not equipped to measure troponins and fractionated creatinine kinase. It can be elevated in cases of hepatocellular damage. 33.

34.

40

The correct answer is C. Status epilepticus can be daunting to manage but requires the same basic steps in managing any emergency. First, secure the airway. Patients who do not appear to have major airway concerns can often get by with a nasal airway or even a simple nasal cannula. This patient, however, has had a prolonged seizure of unknown length and has secretions and blood in his oropharynx. The airway needs to be managed. Medications should be started as soon as the airway is stabilized. The first-line treatment is intravenous lorazepam, which has excellent central activity and a long active half-life in the CNS. Other medications include diazepam, although oral (choice A) is usually not an option for status epilepticus patients; fosphenytoin; rectal diazepam (choice E); and, for resistant seizures, phenobarbital (choice D), midazolam, and propofol. EEG monitoring (choice B) should be started as soon as possible but is not necessary to guide early management of status epilepticus. The correct answer is B. The first treatment for symptomatic bradycardia is atropine. Peri-infarct patients have a high degree of counter-regulatory vagal tone, which can result in hypotension and bradycardia, and will likely respond to atropine. If there is no response to atropine, then transcutaneous (choice D) or transvenous (choice E) pacing is appropriate. In general, transvenous pacing, which requires less energy to be delivered and is significantly more comfortable, is the preferred pacing modality. However, a transcutaneous pacer can be placed quickly and is more useful as an acute treatment. An emergency revascularization procedure, such as coronary artery bypass grafting (choice A), is not necessary until less invasive procedures have failed. Phenylephrine (choice C) is not an appropriate agent for this patient. It results in vasoconstriction without

any beta-mediated increase in heart rate. The increased peripheral resistance can be expected to reflexively lower, not increase, this patient’s heart rate. 35.

The correct answer is C. This patient has glucose-6phosphate dehydrogenase (G6PD) deficiency, a critical enzyme in the hexose monophosphate shunt. The hexose monophosphate shunt serves to protect hemoglobin from oxidative stress. As such, patients are at risk for hemolysis from oxidative damage, which can be induced by infection, fava beans, physical stress, and a variety of drugs, of which sulfa is notorious. Abnormal cellular membrane proteins (choice A) can cause hemolytic anemia. Well-described conditions caused by abnormal cell membranes include paroxysmal nocturnal hemoglobinuria and hereditary spherocytosis. This patient’s erythrocytes lack an enzyme but do not have grossly abnormal cellular membranes. Medications can induce hemolysis through a variety of mechanisms. First, the drug can induce autoantibodies (choice B), such as in drug-induced lupus. Second, the drug can bind with cell membranes, creating a new hybrid antigenic target, or hapten. This hapten complex is recognized as foreign and is destroyed by the immune system as part of a hapten-mediated cellular destruction (choice D). This mechanism is seen with some penicillin allergies. This patient has a classic presentation for G6PD deficiency, which should be suspected first. There is no reason to suspect an inherited hemoglobinopathy (choice E) in this patient. Hemoglobinopathies, such as sickle cell disease and the thalassemias, usually do not present in young adults following the use of sulfa drugs.

36.

The correct answer is C. Inhaled anticholinergics, particularly ipratropium, are the first line treatment of COPD, together with smoking cessation and pulmonary rehabilitation. COPD patients display high parasympathetic tone, which is particularly amenable to muscarinic receptor blocking agents. As ipratropium is a quaternary ammonium compound and thus charged (compared with tertiary agents such as atropine), it dissolves freely in water but is extremely lipophilic. This renders it unable to cross the bronchial barrier, resulting in few systemic side effects. Albuterol (choice A) is another agent available for the treatment of COPD. In head-to-head trials, ipratropium has been shown to be more effective than albuterol in treating COPD. Note that this is the opposite case in the treatment of asthma. COPD patients generally suffer more side effects (partially because they require higher doses) from albuterol than ipratropium. In addition to tachycardia, albuterol can cause pulmonary

Internal Medicine Answers and Explanations

vasodilatation that may worsen ventilation/perfusion mismatching and result in a slight decrease in arterial oxygen saturation. If patients have significant improvement with ipratropium, it should be used as the sole agent. If patients need further treatment, then albuterol may be an appropriate additional agent. Steroid inhalers such as flunisolide (choice B) are worthless in the vast majority of patients with COPD. The neutrophilic inflammation of COPD, in contrast to the eosinophilic inflammation of asthma, does not respond well to inhaled steroids. If patients have a history of asthma or are extremely ill, a trial of inhaled steroids may be appropriate. Oral or parenteral steroids such as prednisone (choice E), however, are of proven benefit in the management of an acute exacerbation of COPD. Were this patient having a severe exacerbation, prednisone would be an appropriate treatment. Home oxygen therapy (choice D) is appropriate only for patients with documented low O2 saturation. The usual criteria is a pulse oximetry saturation of less than 88%, either at rest or with exercise, or a PaO2 of less than 55% (or less than 59% if comorbid conditions are present). 37.

38.

The correct answer is C. This patient has an infectious arthritis. One of the most common causes of infectious monoarticular arthritis in young patients is gonorrhea. Symptoms include migratory arthritis, tenosynovitis, and vesiculopustular skin lesions. Diagnosis can be made through cultures grown on Thayer-Martin medium. Treatment for disseminated gonorrhea infection includes ceftriaxone. Acyclovir (choice A) is used in the treatment of herpetic lesions. Azithromycin (choice B) is used in the treatment of infection with chlamydia, which should be considered in addition to gonorrhea because chlamydia is a common coinfection with gonorrhea. Although ceftriaxone is the preferred agent, ciprofloxacin (choice D) is an alternative regimen that may be used in the treatment of gonorrhea if ceftriaxone cannot be used. A gram-negative agent, Neisseria gonorrhea does not respond well to vancomycin (choice E). The correct answer is D. Given this patient’s immunosuppressant medications and clinical presentation, tuberculosis is the most likely clinical diagnosis. The acid-fast bacilli in the induced sputum further support the diagnosis. Isoniazid, rifampin, pyrazinamide, and ethambutol should be given to this patient for effective management. Given the high

prevalence of multidrug-resistant tuberculosis worldwide, ethambutol also should be administered to achieve the maximum bacterial kill rate in the first 2 months. If the strains are sensitive to the medications administered, ethambutol can be discontinued. Isoniazid (choice A) alone is not given for the treatment of active tuberculosis. It is administered typically by itself for prophylaxis. Isoniazid and rifampin (choice B) may be better than individual therapy. As mentioned, however, prevalence of multidrug-resistant tuberculosis makes it imperative to use the strongest and most effective combination of medications initially. Isoniazid, rifampin, and pyrazinamide (choice C) form the backbone of any first-line therapy of tuberculosis. Given the prevalence of multidrug resistant tuberculosis, however, ethambutol also should be administered initially. Rifampin (choice E) alone is not suitable treatment 39.

The correct answer is E. The patient most likely has the syndrome of inappropriate secretion of antidiuretic hormone (SIADH), in which he is in a sodium-wasting state and is retaining free water. A pulmonary process can induce SIADH. The patient should be placed on free water restriction until the sodium level normalizes gradually. The diagnosis may be confirmed with simultaneous measurement of plasma and urine osmolarity. Administering fluid (choice A) may worsen the hyponatremia by effectively inducing the kidneys to retain free water while wasting sodium. A loop diuretic (choice B) would cause sodium wasting and worsen the hyponatremia. Similarly, salt tablets (choice C) would not correct SIADH. The patient is being treated with antibiotics currently and he has not necessarily failed treatment (choice D).

40.

The correct answer is E. Patients with Wolff-ParkinsonWhite (WPW) syndrome have an accessory pathway (the bundle of Kent) that can allow rapid conduction of atrial impulses to the ventricle. The up-sloping R wave is termed a delta wave and represents pre-excitation of the ventricle. This accessory pathway from the atria to the ventricle also can conduct atrial fibrillation impulses rapidly, resulting in a very rapid tachycardia. The increased risk for sudden death in WPW patients is believed to be secondary to rapid ventricular rates during atrial tachycardia. Understandably, agents are needed that directly suppress the extranodal accessory pathway, rather than just block the AV node, a strategy that is appropriate

41

USMLE Step 2 Assessment Exam

for most forms of atrial fibrillation. Indeed, there is concern that blocking the AV node may increase conduction through the more rapidly conducting bundle of Kent, worsening the rapid ventricular response. Procainamide, flecainide, and propafenone are all reasonable choices to suppress tissue in the accessory pathway. Nodal blocking agents, such as digoxin (choice A), intravenous diltiazem (choice C), or metoprolol (choice D), are not appropriate choices, as they act mainly on the AV node. Emergent cardioversion (choice B) is not necessary if a patient is hemodynamically stable. Because cardioversion carries its own risks and morbidity, it should not be a first-line treatment in a stable patient. If this patient were unstable, then cardioversion would be warranted. After this patient is in a normal sinus rhythm, or at least is rate controlled, radiofrequency ablation of the accessory bundle should be considered. 41.

42

The correct answer is B. This patient has Wilson disease, a defect in biliary copper excretion that results in copper accumulation and toxicity in a variety of organs. Wilson disease should be considered in any young individual with new-onset psychiatric and neurologic dysfunction. The initial presentation is often insidious, and patients are commonly misdiagnosed with ADHD when neuropsychiatric symptoms predominate, or with autoimmune liver disease when hepatic dysfunction is the presenting picture. The most common initial neurologic signs are usually behavioral changes, dysarthria, and dysphagia. Psychosis and depression are common behavioral changes. It is notable that this patient has a KayserFleisher ring, a ring around the iris of dense brown or green copper deposition that is a pathognomonic sign of Wilson disease. The massive, total-body copper overload results in increased urinary excretion of copper. Laboratory diagnosis involves finding increased urinary copper along with decreased ceruloplasmin. Equivocal tests can be clarified with a liver biopsy, which should show increased amounts of cellular copper. Total serum copper concentration (choice A) is usually reduced because there is a deficiency of ceruloplasmin. However, the free copper not bound to the circulating copper binding protein is increased, not decreased (choice D). Ceruloplasmin levels are decreased, not increased (choice C). The genetic defect responsible for Wilson disease results in a failure of hepatic incorporation of copper into apoceruloplasmin, the combination of which forms ceruloplasmin. Low ceruloplasmin levels are an expected finding in Wilson disease, though this test is not specific for the condition.

Hepatic concentrations of copper are markedly increased, not reduced (choice E). As copper overwhelms the liver, which is unable to excrete the substance, cellular damage and eventual necrosis occurs. Free copper is “leaked” into the bloodstream and eventually deposits in other organs. 42.

The correct answer is A. This patient has osteoarthritis, which is the most common joint disease. Osteoarthritis, unlike rheumatoid arthritis, is not an inflammatory disease. The initial therapy should include weight loss, exercise, and acetaminophen. Joint arthroplasty (choice B) is typically reserved for cases in which aggressive medical treatment has been unsatisfactory, especially if the patient’s quality of life has been decreased. Methotrexate (choice C) and prednisone (choice D) may be used in the management of rheumatoid arthritis, not osteoarthritis. Analgesics are typically needed to relieve the pain (choice E).

43.

The correct answer is E. This patient is a high-risk patient who almost certainly has coronary artery disease and should be treated as such. Noninvasive testing is not required to determine the treatment goals of this patient. Aggressive risk reduction with a focus on tight glucose control, meeting tight lipid goals (LDL cholesterol level of less than 100 mg/dL), reinforcing smoking cessation, and so on are all appropriate, regardless of what may show up on noninvasive testing. In general, low-risk and high-risk asymptomatic patients do not require testing to determine their risk for coronary artery disease. For low-risk patients, the positive predictive value of a test is too low, whereas high-risk patients warrant treatment regardless of the results of the test. Coronary computed tomography (choice A) is a relatively new technology that looks for calcium deposits in the coronary arteries. The area and density of calcium are used to generate a coronary calcium score that correlates with the extent of atherosclerosis, though not necessarily the patient’s prognosis. There are no clear guidelines on the use of this modality. Blue Cross Blue Shield finds the technique experimental and does not recommend it, whereas the American College of Cardiology and American Heart Association state that coronary CT can be used on “selected” intermediate risk patients. It can be expected that this patient has some calcium in his arteries; what to do with a quantified version of this information is unclear. Electrocardiographic exercise testing (choice B) should be used on intermediate risk patients to further risk-stratify

Internal Medicine Answers and Explanations

them. Unfortunately, despite its wide use, EKG-stress testing has a sensitivity that ranges from 25 to 75% in asymptomatic populations. Given this patient’s numerous risk factors, including baseline EKG abnormalities, this test is unlikely to provide much additional useful information. This patient should be treated as if he has coronary artery disease. Exercise-stress echocardiography (choice C) and myocardial perfusion scintigraphy (choice D) have a higher sensitivity but are expensive, not always available, and have been studied mainly in symptomatic patients. Were this patient to have cardiac symptoms (which he has a high likelihood of having in the near future), these would be excellent tests to determine his near-term risk for a major event and to determine if invasive testing and treatment (i.e., cardiac catheterization) are appropriate. At this point, however, further testing is not necessary to determine this patient’s cardiac risk and treatment goals. 44.

The correct answer is B. This patient has an empyema, a complicated parapneumonic effusion. Given the evidence of gross pus and concerning findings on pleural fluid analysis (a low pH and glucose), this patient needs drainage. The most effective treatment is by way of chest tube, as repeat thoracenteses (choice E) are rarely effective if loculations (evidenced by the lack of layering on decubitus films) are present. Broad-spectrum antibiotics (choice A) alone are unlikely to resolve this infection. Likewise other antibiotics such as ciprofloxacin (choice C) are unlikely to provide much benefit without definitive drainage. Pleurectomy (choice D) is a dramatic intervention that involves removal of one or more of the ribs and stripping of the pleura. It is rarely used nowadays and is not appropriate for the treatment of the vast majority of effusions.

45.

The correct answer is C. This patient likely has CNS toxoplasmosis, a common opportunistic infection occurring in up to 40% of patients with HIV, and which is characterized by ring-enhancing lesions on CT scan. The treatment of choice is pyrimethamine, sulfadiazine and leucovorin. Clindamycin is used for patients with sulfa allergies. In addition to imaging with CT or MRI, antibody titers can be used to make the diagnosis. More than 15% of patients have negative titers, however, presumably because of B-cell dysfunction in advanced HIV infections. Amphotericin B (choice A) is appropriate for fungal infections but not for toxoplasmosis. Toxoplasmosis is an obligate intracellular parasite and does not respond to antifungal agents.

There is always a debate as to whether to give glucocorticoids (choice B) when patients have mass lesions. In general, steroids should not be given to patients with likely toxoplasmosis. If a B-cell lymphoma is present, also common in this population and that sometimes has a similar imaging appearance, it will (temporarily) shrink in response to the steroids, confusing the diagnosis. If the diagnosis is unclear and the patient’s situation dire, a trial of steroids while the diagnosis is confirmed may be appropriate. Surgical resection (choice D) is not used for toxoplasmosis, and consultation is not appropriate at this time. Trimethoprim/sulfamethoxazole (choice E) is not used for toxoplasmosis. It is, however, the main treatment for Pneumocystis carinii pneumonia, a common opportunistic infection in this population of patients. 46.

The correct answer is C. There are few interventions that prolong life and preserve lung function among COPD patients. Long-term home oxygen can improve symptoms of COPD and can prolong life in patients with severe COPD. Among those with earlier manifestations of the disease, the only truly effective treatment is smoking cessation. Shortly after smoking cessation, the rate of lung decline returns to that of a nonsmoker. If patients continue to smoke, they annually lose two to three times the lung capacity of nonsmokers, a decline in FEV1 of approximately 50 to 60 mL per year. Inhaled beta-agonists (choice A) and inhaled cholinergic antagonists (choice E) are a cornerstone of treatment for severe COPD. Cholinergic antagonists are more effective in relieving symptoms of COPD, but as the disease progresses, patients often require both forms of treatment. Although both improve symptoms, neither preserves lung function, improves survival, or in any way alters the course of the disease. Inhaled glucocorticoids (choice B) are not effective for the vast majority of COPD patients. For ill patients with an exacerbation of COPD, oral steroids can improve symptoms and reduce the length of exacerbation. Lung reduction surgery and lung transplantation (choice D) are treatments for patients with end-stage COPD. Both interventions improve quality of life, but neither provides a mortality benefit.

47.

The correct answer is E. Weakness and fatigue have a broad differential, but myasthenia gravis, particularly in a young woman with ocular and bulbar weakness, needs to be on the differential diagnosis list. The Tensilon test, which involves administering an intravenous dose of edrophonium and watching for improvements in strength, neatly elucidates the pathophysiology of the

43

USMLE Step 2 Assessment Exam

disease. Myasthenia is caused by an antibody-mediated autoimmune attack directed against nicotinic acetylcholine receptors. The antibodies initially competitively block the receptor, which is absorbed eventually by endocytosis and destroyed in a lysosome. An increase in acetylcholine can overcome the competitive inhibition and result clinically in an increase in strength. Edrophonium is a short-acting cholinesterase inhibitor that results in an increased amount of acetylcholine available at the neuromuscular junction. The transient improvement in strength is clinically apparent, with patients suddenly being able to tighten their grip or to hold their upward gaze. This should be seen immediately and from the first dose. If there is no immediate effect from the first dose of edrophonium (choice B), a different diagnosis should be suspected. The medication has a short half-life with a quick onset of action; gradual improvement (choice A) is unlikely. Long-acting cholinesterase inhibitors are available, though they are not usually the mainstay of therapy. The patient is expected to have an improvement of symptoms with edrophonium. If there is a reproduction of severe symptoms of weakness (choice C), then the patient is suffering from a cholinergic crisis in which an excess of acetylcholine results in a depolarizing neuromuscular blockade. This can occur in patients who are over treated for their myasthenia, in which case the Tensilon test is useful for distinguishing an exacerbation from a cholinergic crisis. Edrophonium indirectly increases parasympathetic tone. A temporary increase in pulse and sympathetic tone (choice D) is unlikely. 48.

44

The correct answer is A. Cirrhosis and the resulting portal hypertension often result in gastrointestinal bleeds. Beta-blockers, such as propranolol, are excellent prophylactic therapy for portal hypertension. Nonselective beta-blockers block mesenteric artery dilatation, resulting in unopposed alpha-mediated constriction. This reduced portal inflow, combined with a small, direct reduction in portal venous pressure, reduces the potential for portal hypertension-related gastrointestinal bleeds. In general, portal hypertension can cause gastric or esophageal varices that may produce dramatic, brisk bleeds. Additionally, small ectatic gastric veins can rupture or cause mucosal ischemia, creating less dramatic, slower bleeds. This condition is termed portal hypertensive, or congestive, gastropathy. The only proven treatment is beta-blockade, which is believed to be significantly better than no additional treatment (choice C).

As the etiology of this patient’s bleed is from vascular irregularities, acid-reducing treatment with H2 blockers (choice B) or proton pump inhibitors (choice D) is unlikely to be effective. Treatments with thermal coagulation, injection therapy, acid-reducing treatment, sucralfate, and surgical resection (choice E) have not proven effective in preventing rebleeding. 49.

The correct answer is E. It is important to recognize that this patient has had a splenectomy, a definitive treatment for the anemia and hyperbilirubinemia associated with hereditary spherocytosis. Mild cases are treated with supplemental folic acid (choice C) and occasional transfusions. Splenectomy does not treat the spherocytes (which are still present post surgery), but treats the hemolytic anemia that results from splenic destruction of these atypical cells. There are numerous clues that this patient is asplenic, including most obviously a surgical scar, but also a normal hemoglobin level and the presence of Howell-Jolly bodies on the peripheral blood smear. It is important to make sure this patient has had a pneumococcal vaccine in the past, as splenectomized patients are at risk for overwhelming sepsis. Further, it sounds like this patient may have had his operation before the current multivalent formulation was available (which became widespread in the early 1980s). If it is unclear if the patient has had a pneumococcal vaccine, he should receive one now. If it can be established with certainty that the patient had a vaccination, then no additional treatment (choice D) is necessary. Corticosteroids (choice A) play no role in the treatment of spherocytosis. This is a hereditary defect in the cell wall membrane, not an autoimmune phenomenon. Dental antibiotic prophylaxis (choice B) is necessary for patients with valve abnormalities, not splenectomies. Current recommendations for splenectomized patients are to not give prophylactic antibiotics before dental procedures unless there is another reason warranting antibiotics.

50.

The correct answer is D. Medications need to be started cautiously and followed closely in elderly patients. Antihypertensives often are started in elderly patients with systolic hypertension, who, although benefiting immensely from treatment, are also highly susceptible to drug side effects. Hydrochlorothiazide can cause a hypokalemia that manifests as weakness and muscle cramps, or a hyponatremia that presents as lethargy, confusion, and weakness. Additional side effects include increased uric acid, increased LDL, and hyperglycemia. Captopril (choice A) is an angiotensin-converting enzyme (ACE) inhibitor. Major side effects include hyperkalemia, chronic cough, and acute renal insufficiency.

Internal Medicine Answers and Explanations

Clonidine (choice B) is a centrally acting antihypertensive known to cause drowsiness, depression, and dry mouth. Although it can cause fatigue, it is unlikely to cause muscle cramps. Adverse events that do not require a gradual depletion of electrolytes generally occur shortly after starting treatment. Felodipine (choice C) is a calcium channel blocker that is often used to treat systolic hypertension. Its most common side effect is constipation, though in high doses it can cause confusion, drowsiness, hypoglycemia, and cardiovascular collapse. Metoprolol (choice E) and other beta-blockers do not commonly result in electrolyte abnormalities, the most likely explanation for this patient’s symptoms. Major adverse effects include a low heart rate and symptoms of orthostasis, bronchospastic airways, acute exacerbation of heart failure, and facilitation of depression, fatigue, and sexual dysfunction. 51.

The correct answer is B. This patient has silica dust exposure and has a clinical picture consistent with silicosis and progressive, massive fibrosis. Patients who work in quarries or around industrial stone grinding, sandblasting, mining, and the like are at risk for pneumoconiosis caused by fine particles of silica. Patients with continued exposure, such as this man who has worked in the family business, will have progressive fibrosis, which shows on radiograph as multiple nodules with larger nodules occurring in the apical aspects of the lungs. Pathologic examination reveals a fibrotic, “honeycombed” lung. There is no reason to suspect a chronic, postobstructive pneumonia (choice A). A more significant productive cough, evidence of consolidation on exam or radiograph, or fevers might suggest the presence of infection, none of which were present. Silicosis does place the patient at an increased risk for tuberculosis. Again, you would expect more in the way of associated symptoms. Severe coughing, purulent sputum, hemoptysis, and cavitating lesions would be more suggestive of tuberculoma, in which case you might find masses composed of acid-fast organisms and necrotic tissue (choice C). There is no reason to think this patient has small-cell lung cancer (choice D). Although he is a smoker, the bilateral lung findings are consistent with an interstitial disease rather than a rapidly spreading small cell malignancy. Asbestos exposure is the major risk factor for mesothelioma, which can present as a pleural-based tumor with encroachment into lung parenchyma (choice E). This patient’s clinical presentation, exposure history, and

chest radiograph are more consistent with interstitial lung disease such as silicosis. 52.

The correct answer is C. This patient likely has cryoglobulinemia, a well-described extrahepatic manifestation of hepatitis C. These abnormal antibodies that precipitate and induce a vasculitis typically manifest as palpable purpura and renal involvement, as seen in this patient. Cryoglobulinemia can be severe, necessitating combined liver and renal transplant in some cases. Given the strong association between hepatitis C and this form of vasculitis, patients with an unexplained vasculitis, particularly if manifesting as palpable purpura, should be tested for hepatitis C. Hepatitis A (choice A) is an acute, self-limiting disease and is not associated with long-term systemic conditions. It is not associated with any common forms of vasculitis. Hepatitis B (choice B) can cause polyarteritis nodosa, a small and medium vessel vasculitis. In general, this vasculitis is less likely to present with palpable purpura and more likely to have diffuse, systemic involvement than cryoglobulinemia. Again, palpable purpura in a patient with liver disease is highly suggestive of hepatitis C. Hepatitis D (choice D) can cause fulminant hepatic failure when it occurs as a coinfection with hepatitis B. It does not cause these symptoms of vasculitis. Hepatitis E (choice E) is rare in the United States and generally causes an acute, self-limited hepatitis, though it may cause fulminant hepatic failure in pregnant women.

53.

The correct answer is C. This patient likely has autoimmune thrombocytopenia, also known as idiopathic thrombocytopenic purpura (ITP). The diagnosis requires excluding other causes of thrombocytopenia, confirming that this is a case of isolated thrombocytopenia (which dramatically narrows the differential diagnosis), and, basically, seeing how the patient responds to therapy. Given this patient’s young age and that she is not taking any other medications, it is unlikely that she has other potential causes of thrombocytopenia, such as drug-induced thrombocytopenia or malignancy. The normal peripheral smear rules out other causes such as platelet clumping (“pseudothrombocytopenia”) or microangiopathic conditions. The most appropriate next step is to give this patient a trial of steroids and see how she responds. Some patients will respond completely; others will have a response but will not be able to be successfully tapered from highdose steroids. These patients may require splenectomy (choice D) and/or intravenous immunoglobulin

45

USMLE Step 2 Assessment Exam

(choice E). Patients who still fail therapy may be tried on immune modulating agents such as azathioprine or cyclophosphamide. Measuring platelet autoantibodies (choice A) seems to make intuitive sense. Unfortunately, this test is neither sensitive nor specific, and is not required or expected to confirm the diagnosis of ITP. Currently, the American Society of Hematology ITP practice guidelines do not recommend measuring platelet autoantibodies because the results can delay and confuse the diagnosis. A bone marrow biopsy (choice B) is not necessary in most patients with isolated thrombocytopenia because it is extremely rare to have a condition that results in this isolated abnormality. However, patients over age 60 are much more likely to have myelodysplastic syndrome, which may present like this. As such, patients over 60 and those with abnormalities on the blood smear suggestive of hematoproliferative disorders should have bone marrow biopsies. This patient, however, does not warrant this invasive procedure. 54.

46

The correct answer is C. Not all cases of syncope require an extensive workup. If the patient has a clear cause—in this case a vasovagal episode potentiated by dehydration—further expensive and low-yield diagnostic tests are not necessary. Vasovagal syncope, in which an exaggerated parasympathetic response causes increased vagal tone, is the most common cause of syncope and is often worse in patients with volume depletion. It is not uncommon to feel ill for a short time before and after the event; this needs to be differentiated from the prodrome and postictal confusion that surrounds seizures. An exercise stress test (choice A) is appropriate if ischemia is suspected. Given that this patient is young and has no significant past medical history, a stress test is not appropriate for his syncopal episode. A head CT (choice B) is recommended for all newonset seizures that present in the emergency room. This patient has no evidence of having had a seizure. A signal-averaged electrocardiogram (choice D) amplifies and filters the ECG signals in order to detect low-amplitude, high-frequency signals in the terminal portion of the QRS complex (so-called “late potentials”). These signals are characteristic of subtle ventricular electrical abnormalities that predispose to ventricular tachycardia. The ECG is not an appropriate study in this patient, who is unlikely to have a cardiac cause of his syncope. Upright tilt testing (choice E) can be used to provoke a vasovagal response. It is useful in patients with recurrent syncopal episodes who do not have evidence of heart disease or seizures. The results are often not reproducible, making this test somewhat difficult to interpret.

55.

The correct answer is D. This patient has scurvy, which results from vitamin C deficiency. Collagen synthesis depends on adequate stores of vitamin C. Without adequate vitamin C, dysfunctional collagen results in capillary fragility, poor wound healing, and abnormal hair development. Patients will often be subject to gum disease, and may lose teeth as the collagen mesh of their gums breaks down. The loss of teeth can further worsen the poor dietary intake because eating fruits and vegetables high in vitamin C becomes more difficult. Niacin (choice A) deficiency can cause pellagra, which presents with dermatitis, dementia, and diarrhea, among other symptoms. It is extremely rare in developed countries, where it tends to occur mainly among alcoholics. In less developed countries, where diets may consist of mainly low-niacin-content foods (cereals, corn, and sorghum), it is still endemic. Common findings include symmetric hyperpigmentation, a red tongue, diarrhea and vomiting, and neurologic symptoms including anxiety, disorientation, dementia, and encephalopathy. Findings associated with pyridoxine deficiency, commonly known as vitamin B6 (choice B), include peripheral neuropathies, seizures, dermatitis, and glossitis. Because pyridoxal phosphate is involved in heme synthesis, a microcytic anemia is also seen. This is a rare deficiency but may be seen with drugs that are pyridoxine antagonists, such as isoniazid. Vitamin B12 (choice C), or cobalamin, deficiency can cause a macrocytic anemia and neurologic changes, and is classically associated with pernicious anemia. Dietary B12 deficiency is rare but is more likely to be present in the elderly or those on fad diets. Vitamin K (choice E) is a key vitamin involved in the production of liver-dependent clotting factors. While a vitamin K deficiency may present as bleeds and anemia, one would expect an elevated prothrombin time from reduced production of factors II, V, VII, and X.

56.

The correct answer is E. This patient has neurosyphilis and needs a significantly higher dose of penicillin for CSF penetration. The treatment of neurosyphilis would be very difficult with the current regimen. Currently, the recommendations are to prescribe 3 to 4 million units of a short-acting penicillin for a 10- to 14-day course. Ceftriaxone (choice A) has a 23% failure rate in the treatment of neurosyphilis. Doxycycline (choice B) is not used in the treatment of neurosyphilis. It is used in the treatment of gonorrhea. If used for treating latent syphilis, the duration is for 28 days.

Internal Medicine Answers and Explanations

Bone scans (choice A) are more sensitive than plain films in detecting metastatic involvement and provide information about the entire skeleton in one exam; however, they are not as sensitive and specific in detecting spinal metastases as an MRI, and bone scans cannot delineate soft tissue and spinal cord anatomy. Further, osteolytic metastases produced by multiple myeloma and lymphoma are not detected by bone scan. A CT scan of the brain (choice B) might possibly demonstrate a cause for this patient’s weakness, but the tenderness in the spine makes it considerably less likely to be in the brain than in the cord. Lumbar puncture (choice C) would rule out carcinomatous meningitis, but that is farther down the differential than cord compression or brain metastases. Although checking the patient’s prostate-specific antigen (choice E) would be helpful, it would take some time for the result to return. Of much greater importance would be diagnosing and managing his current problem of cord compression, which could result in permanent loss of neurologic function.

Penicillin benzathine (choice C) is a long-acting penicillin. This dosage is used to treat syphilis that has been present for >1 year. The treatment should be for not less than 10 days (choice D). 57.

58.

The correct answer is D. This patient most likely has pulmonary hypertension from chronic hypoxia. A clue to this diagnosis is the fact that she has been snoring at night and is moderately obese, thus raising the likelihood of sleep apnea. Her chronic hypoxia makes her susceptible to pulmonary hypertension and an elevated right ventricular systolic pressure. This diagnosis can be made with a sleep apnea test or a polysomnography test. This patient is denying symptoms that are consistent with coronary artery disease. She has no chest pain. There is little reason to believe that she would have a positive stress test (choice A). Similarly, a left heart cardiac catheterization (choice B) would be indicated in a patient with a positive stress test or symptoms consistent with an acute coronary syndrome. A lung biopsy (choice C) would be indicated if all other workup is negative and interstitial lung disease is suspected. A right heart cardiac catheterization (choice E) would confirm the diagnosis of pulmonary hypertension by measuring the pressures in the various cardiac chambers. This invasive test can be deferred at this point, however. The correct answer is D. The patient has metastatic spinal cord compression, which occurs in patients with lung, prostate, and breast cancer, as well as with hematologic malignancies such as multiple myeloma and lymphoma. Pain accompanies spinal cord compression in the majority of cases. The site of compression can usually be localized to the site of back or neck pain. Weakness is the second most common symptom. Other symptoms and findings include overflow incontinence, flaccid paralysis of the lower extremities, absent reflexes, saddle anesthesia, and impotence. Plain films are useful in evaluating cord compression due to bony involvement of the spine or compression fracture, but they cannot show the actual spinal cord itself or any soft tissue masses. Magnetic resonance imaging has a very high diagnostic accuracy (95%) in detecting cord compression. It can image soft tissue anatomy in great detail, and multiple levels of cord impingement can be seen in one exam.

59.

The correct answer is B. This patient has a high clinical suspicion for a pulmonary embolus. The V/Q scan essentially confirms the diagnosis. Oral contraceptive pills in addition to smoking place her at an increased risk for thromboembolic disease. The most common symptoms of a pulmonary embolism include tachypnea and tachycardia. It is also a cause of shock, and a massive pulmonary embolism can result in the loss of blood pressure. This patient has a stable blood pressure at this time. Thus, placing her on heparin immediately would be appropriate to prevent the clot burden from increasing. Aspirin (choice A) would inhibit platelet aggregation but would not be effective in treating this patient. Streptokinase (choice C) would be used if the pulmonary embolism were to be lysed immediately. This would be indicated if the patient was hemodynamically unstable with a low blood pressure. Warfarin (choice D) would be appropriate as a longterm choice for anticoagulation in this patient. Ultimately, this patient would be switched to warfarin and taken off heparin. Typically, the duration of coumadin would be 6 months, at which time the patient would be reassessed. An inferior vena cava filter (choice E) would be indicated if the patient developed recurrent clots on warfarin.

60.

The correct answer is B. Activated charcoal is a fine, nonabsorbable powder that has a network of interconnecting pores capable of adsorbing different toxins

47

USMLE Step 2 Assessment Exam

quickly. It is useful for a variety of substances and should be given in most cases of acute poisoning, as it is extremely rare to be 100% sure of what a patient may have ingested. Unfortunately, not all substances are well absorbed by charcoal. Small, highly ionized chemicals, such as metals, electrolytes, acids, and alkali are not well absorbed. As such, iron overdose or lithium toxicity is not affected by charcoal. Further, hydrocarbons and alcohols also are unaffected by charcoal, making it impossible to treat an intoxicated individual with charcoal. Acetaminophen (choice A), amitriptyline (choice C), aspirin (choice D), and digoxin (choice E) are all adsorbed by the micropores of activated charcoal. In general, the sooner the administration of charcoal, the greater the likelihood of stopping drug absorption. Charcoal should not be withheld arbitrarily, however, because of a presumed timeline of drug absorption. Many medications come in delayed release forms or have gut dysmotility effects (for example, the anticholinergic effects of amitriptyline) that can delay systemic absorption. 61.

62.

48

The correct answer is A. Crohn disease, and other diseases that cause fat malabsorption such as sprue and pancreatic disease, can result in fat-soluble vitamin deficiency. The fat-soluble vitamins are D, E, A, and K (DEAK). Vitamin A deficiency can result in nightblindness, conjunctival hyperkeratosis (Bitot spots), and a variety of dermatologic manifestations. Vitamin B (choice B) encompasses a variety of vitamins, with a host of different deficiencies. These are rarely seen in developed countries because vitamin B is ubiquitous in the food supply. Further, this patient is not at risk for deficiencies in water-soluble vitamins. Vitamin D (choice C) deficiency manifests as osteoporosis in adults and rickets in children, as vitamin D is necessary for effective calcium absorption. Vitamin E (choice D) deficiency is extremely uncommon but may manifest as ataxia and hemolytic anemia. Vitamin K deficiency (choice E) can occur in newborns and in patients on total parenteral nutrition or longterm antibiotics (which can alter vitamin K–producing gut flora). A prolonged prothrombin time or frank bleeding may be present. The correct answer is D. What began as simple urinary tract obstruction has become the dreaded urologic emergency of obstruction plus infection. In addition to antibiotics, emergency decompression of the urinary tract above the obstruction is needed. Antibiotics alone (choice A) will not be enough; a nephrostomy is needed.

Alkalinization of the urine (choice B) can dissolve uric acid stones. Uric acid stones are not calcified, so she does not have that kind of stone. Furthermore, she is facing an emergency that requires immediate resolution. Endoscopic removal of the stone (choice C) sounds appealing, but it is absolutely contraindicated. Manipulation and instrumentation of the urinary tract in the present situation would lead to systemic sepsis. It is too late to remove the stone now. After decompression, it may be okay to do it. The same could be said for shock wave lithotripsy (choice E). It might be acceptable to do it once the acute problem is over, but not right now. 63.

The correct answer is C. In the presence of progressive, unstable angina, he should be evaluated for potential coronary revascularization before major noncardiac surgery is performed. If he is a suitable candidate, both operations could be performed simultaneously, or he could undergo the revascularization before his aneurysm is fixed. Echocardiograms (choice A) are used to work up suspected morphologic abnormalities of the heart. That procedure would not reveal the necessary details about the coronary circulation. Exercise tolerance test and thallium scan (choice B) are indicated to unmask silent coronary disease in highrisk individuals. It would be hazardous to subject a patient with unstable angina to this particular test. If he were in congestive failure, digitalis and diuretics (choice D) could be part of his treatment, but he has no evidence of failure at this time. The standard time interval to wait to reduce operative risk after an MI is 6 months. This man is already 8 months out. Waiting a full year as proposed in choice E adds no further advantage.

64.

The correct answer is C. This patient has acute Guillain-Barré syndrome (GBS), an acute demyelinating polyradiculopathy that is often associated with a recent infection. The manifestations include ascending weakness, areflexia, and an increase in cerebrospinal fluid protein. Treatment is typically with plasmapheresis or intravenous immunoglobulin. Glucocorticoids (choice A) and interferon-beta (choice B) are not effective in the treatment of acute GBS. They are often used in multiple sclerosis. Pyridostigmine (choice D) and thymectomy (choice E) are not effective in the treatment of acute GBS. They are treatments for myasthenia gravis.

Internal Medicine Answers and Explanations

65.

66.

67.

The correct answer is D. The patient’s electrocardiogram shows that she has myocardial ischemia of the left ventricle. The diabetes and hypercholesterolemia are risk factors for her coronary artery disease. Treatment must be initiated to decrease her myocardial oxygen demand. Costochondritis (choice A) and musculoskeletal pain (choice B) are not associated with electrocardiographic changes. Myocardial infarction (choice C) is typically associated with either ST elevations or elevated serum markers, neither of which is mentioned in this vignette. The electrocardiographic findings are consistent with ischemia, not a pulmonary embolism (choice E). The correct answer is C. The most common cause of microcytic anemia is iron deficiency, and a low serum ferritin confirms that diagnosis. This man has probably been chronically losing blood, and the most likely source is the gastrointestinal tract. Cancer of the right side of the colon is a good bet, and once occult blood is found in the stool, a colonoscopy should follow. Coombs test (choice A) is used for the diagnosis of hemolytic anemia, which is normocytic and does not deplete iron stores. Hemoglobin electrophoresis (choice B) is used for the diagnosis of thalassemia. The anemia in that disease is indeed microcytic, but the patient would have normal iron stores (normal serum ferritin). The Schilling test (choice D) has a role when patients have macrocytic anemia, as it helps differentiate pernicious anemia from intestinal malabsorptive disorders. Anemia caused by renal insufficiency is normocytic, in which case serum creatinine (choice E) would be a useful test. The correct answer is A. The history is suggestive of episodes of hypoglycemia, possibly triggered by an insulinoma. Fasting should induce the symptoms, at which time one could demonstrate the presence of hypoglycemia and high levels of insulin and C peptide. Resolution of the symptoms by glucose administration would then complete the diagnostic “Whipple triad.” CT scan of the pancreas would follow. Tumors involving the frontal lobe may produce behavioral changes, and funduscopic examination (choice B) and MRI (choice D) would be helpful to diagnose that problem. In that case, though, the behavioral changes should be accompanied by persistent headaches, visual changes, and possibly olfactory deficits (Foster Kennedy syndrome). There would be no correlation with fasting or resolution by eating.

Liver failure is a potential cause of hypoglycemia, but the latter would not occur until liver function is severely compromised. The patient would have overt signs of liver failure, rather than a minor, occult deficiency that would be discovered by liver function tests (choice C). If the indicated tests were to show that his symptoms are indeed caused by hypoglycemia, at which time blood levels of insulin were found to be high while the C peptide was low, self-administration of insulin should be suspected. It would be at that point that psychiatric evaluation (choice E) would be indicated. 68.

The correct answer is C. The presentation is highly suggestive of Zollinger-Ellison (Z-E) syndrome. Further studies after her serum gastrin level is determined might include secretin stimulation test, if needed for confirmation, and CT scan of the pancreas to locate the gastrinoma. Secretory diarrhea in this setting is part and parcel of Z-E syndrome, and pathology in the intestinal mucosa need not be sought (choice A). Biopsy of the ulcer (choice B) is dangerous and unnecessary. Biting into an ulcer that recently bled is an invitation to rebleeding, and ascertaining if the ulcer is benign or malignant is indicated in the case of gastric ulcers but is not pertinent for duodenal ulcers. At some point, serum calcium (choice D) needs to be measured. Gastrinomas may coexist with parathyroid adenomas. It would not be the first test needed, however, before the presence of Z-E has been established. Upper gastrointestinal series with barium (choice E) does not add any information to that already provided by the endoscopy.

69.

The correct answer is B. This man has massive gastrointestinal bleeding, and the nasogastric aspirate indicates that it is coming from beyond the ligament of Treitz. Approximately 70% of patients who bleed from a source distal to that landmark do so from diverticulosis. Approximately one quarter of those present with massive bleeding. The offending diverticula are usually in the right colon and thus are not visible to a proctosigmoidoscopic examination. The usual patient with this presentation is old enough to have diverticulosis. All the numbers quoted fit this particular case. Colon cancer (choice A) also occurs in older people. It is also a frequent source of bleeding, but typically the bleeding is not massive. Furthermore, the left side of the colon hosts cancers more frequently than the right. This man could be bleeding from a colonic cancer, but obviously that would not be the most likely situation.

49

USMLE Step 2 Assessment Exam

Duodenal ulcer (choice C) has been excluded by the nasogastric aspirate. Upper gastrointestinal sources always have to be considered when people bleed per rectum (a common dictum says that a common cause for lower gastrointestinal bleeding is in fact upper gastrointestinal bleeding), and that is why the nasogastric aspiration is done. Once one sees green fluid without blood, however, one stops thinking of upper gastrointestinal lesions. Inflammatory bowel disease (choice D) can bleed, but that source is way down on the list of possibilities. Portal hypertension (choice E) can produce massive bleeding. Typically, however, it is by way of esophageal varices (which have been ruled out by the nasogastric aspirate) or hemorrhoids (which have been ruled out by direct examination of the anorectal area). 70.

71.

50

The correct answer is A. Although this patient has the clinical presentation of diverticulitis, the key point is that the patient was recently treated with gentamicin and is therefore at risk for nephrotoxicity. This takes the form of acute tubular necrosis and may either be oliguric or nonoliguric. Patients with the nonoliguric form generally recover their renal function if the aminoglycoside is discontinued and the patient is adequately hydrated. Diverticulitis (choice B) has no direct effect on renal function. Theoretically, patients with diverticulitis without septic shock may develop renal insufficiency, but there is no evidence of that scenario here. Left ureteral obstruction (choice C) may occur in a patient with sigmoid diverticulitis but would not cause renal insufficiency unless the patient also had right kidney dysfunction. Pyelonephritis (choice D) is not described by the physical and CT scan findings in this patient, and unilateral pyelonephritis would not cause renal insufficiency. There is no evidence by physical examination that this patient is in shock (choice E). The correct answer is D. This previously healthy young woman has significant systolic and diastolic hypertension. The classic clue here is an upper abdominal bruit, which suggests the diagnosis of renovascular hypertension. In a young woman, the diagnosis of renovascular hypertension is usually due to fibromuscular dysplasia, which is usually unilateral and unifocal. Although aortic dissection (choice A), coarctation of the aorta (choice B), hypertrophic cardiomyopathy (choice C), and Takayasu arteritis (choice E) are all associated with hypertension, they do not produce the upper

abdominal bruit heard here. Furthermore, choices A, B, and E will generally produce a discrepancy in blood pressure between the arms or between the arms and legs. There is no evidence on exam of a hypertrophic cardiomyopathy, i.e., no murmurs or S4 are described. 72.

The correct answer is E. This case describes a typical presentation of bacterial meningitis. Suggestive of meningitis are the patient’s headache and photophobia. Suggestive of a bacterial cause are the patient’s temperature to 39.5 C (103.1 F), cloudy CSF, and positive Gram stain. Streptococcus pneumoniae is one of the two leading causes of purulent meningitis in this age group. Neisseria meningitidis is the other cause and may occur in epidemics. Encapsulated yeast with hyphae (choice A) is a rare cause of meningitis, certainly in an immunocompetent host. Cryptococcus neoformans, the leading cause in the yeast family, is an oval, budding yeast that does not exist as hyphae. Gram-negative bacilli in pairs (choice B) describes a diverse group of organisms. Typically, these are found in the GI tract and respiratory tract. The two that are more commonly associated with meningitis are Haemophilus influenzae and Escherichia coli. These are the leading causes in infants and children younger than 6 years old. (Group B strep is most common in neonates.) Gram-negative cocci in clusters (choice C) is not a typical description of an organism. Neisseria, the most common Gram-negative coccus, is a diplococcus that occurs in pairs. Gram-positive bacilli in pairs (choice D) include four genera, of which Listeria monocytogenes is commonly associated with meningitis in neonates and immunocompromised hosts.

73.

The correct answer is C. This patient is having an acute myocardial infarction (MI), as evidenced by ST segment elevations on the EKG in the setting of the past medical history. The treatment for acute MI, with either ST segment elevations of more than 1 mm in two contiguous leads or a new left bundle branch block, is immediate reperfusion with either intravenous thrombolytics or coronary balloon angioplasty via coronary artery catheterization. This is the absolute standard of care. Admission to the coronary care unit for exclusion of MI (choice A) is inappropriate because all evidence suggests that the patient is actively infarcting. Emergent coronary artery bypass surgery (choice B) is not done on an emergent basis, but rather the patient would be brought to the cardiac catheterization laboratory for percutaneous intervention.

Internal Medicine Answers and Explanations

Cardioversion (choice D) is not indicated, inasmuch as the patient is not having any unstable arrhythmias associated with the acute MI. A PET scan (choice E) is not done on an emergent basis but to detect residual functional myocardium after an ischemic event. 74.

The correct answer is A. The 5-TU PPD reaction in this patient is considered to be negative because he has no risk factors, is from a low-incidence group, and has a reaction of less than 15 mm. The reaction to 5-TU PPD has to be 15 mm or greater to be considered a positive result in a person with no risk factors who is from a low-incidence group. Therefore, no further management is needed for this patient. Combination therapy such as isoniazid, rifampin, and pyrazinamide (choice B) is used for treating active tuberculosis. This patient has no signs or symptoms to suggest active tuberculosis. Isoniazid alone (choice C), when used for 6 to 12 months as prophylactic therapy in a patient with a positive 5-TU PPD reaction, significantly reduces the risk of tuberculosis. Rifampin alone (choice D) can be used as an alternative in prophylactic therapy (usually in combination with ethambutol) in patients who are potentially infected with drug-resistant organisms. There is no role for these medications in this patient who has a negative result. Bronchoscopy (choice E) is used as a diagnostic tool to confirm active tuberculosis. Because active tuberculosis is not suggested in this case, bronchoscopy has no role in the care of this patient.

75.

The correct answer is C. This patient likely has influenza, though other viral etiologies cannot be ruled out. If a rapid influenza test is available (not common in most clinics), then a “test and treat” strategy is useful. Unfortunately, viral antibody titers (choice A) rise after the acute phase of the illness, thus negating their usefulness in treatment. Most cases of influenza are self-limited. However, if a patient desires treatment, amantadine and the neuraminidase inhibitors (such as oseltamivir) have been proven to reduce the duration of illness by about a day if started within 2 days of symptom onset. If the patient has been ill for more than 2 days, however, then no additional treatment is available (choice E). Empiric macrolide antibiotics (choice B) are not appropriate for this patient, inasmuch as her illness is probably not bacterial. Patients with COPD will often have flares of bronchitis, caused or exacerbated by organisms that are susceptible to macrolides. High-dose vitamin C and zinc lozenges (choice D) have both been proposed as “cures for the common cold.” Unfortunately, in rigorous, controlled studies neither treatment has shown benefit. That said, if a patient wishes to take supplemental vitamin C and zinc, there is little harm to be had and perhaps much to be gained from maximizing the placebo effect of these treatments.

51

BL3163A Printed in USA © 2003

kaplanmedical.com

View more...

Comments

Copyright ©2017 KUPDF Inc.
SUPPORT KUPDF